Определитель имеющий два одинаковых ряда равен: HTTP 404 Resource not found

Содержание

2.3. Свойства определителей

Определители

Первые три перестановки являются четными, поскольку каждая из них содержит четное число инверсий.

Оставшиеся три перестановки являются нечетными, так каждая из них содержит нечетное число инверсий. (См. пример на стр. 22.)

Таким образом,

a1,1 a1,2 a1,3

a2,1 a2,2 a2,3 = a1,1a2,2a3,3 +a1,2a2,3a3,1 +a1,3a2,1a3,2 a3,1 a3,2 a3,3

−a1,3a2,2a3,1 −a1,2a2,1a3,3 −a1,1a2,3a3,2

Эту формулу можно легко запомнить с помощью правила треугольников, которое иллюстрируется нижеприведенным рисунком.

Элементы, стоящие на диагоналях или в вершинах треугольников, основания которых параллельны диагоналям, образуют произведения трех элементов. Если основание треугольника параллельно главной диагонали матрицы, то произведение элементов сохраняет свой знак. Если же основание треугольника параллельно побочной диагонали матрицы, то произведение элементов берется с противоположным знаком.

1.Определитель транспонированной матрицы равен определителю исходной матрицы:

det AT = det A .

Это свойство выражает равноправие строк и столбцов определителя. Доказательство: Свойство вытекает из определения детерминанта – оба детерминанта представляют собой суммы одних и тех же элементов.

2.Умножая строку или столбец определителя на число λ, мы умножаем определитель на это число:

25

Определители

Можно сказать и так: Общий множитель строки (или столбца) можно выносить за знак определителя.

Доказательство: При умножении строки определителя на число один из сомножителей в произведении a1,k1 a2,k2 Kan,kn (−1)P{k1, k2 , K, kn }

умножается на это число. Это число входит в виде общего множителя в каждое слагаемое суммы

det A = ∑a1,i1 a2,i2 Kan,in (−1)P{i1 ,i2 ,K,in } . {i1 ,i2 ,K,in }

3.Определитель изменяет свой знак при перестановке местами любых двух строк (или столбцов) определителя.

Доказательство: По Теореме 1, транспозиция изменяет четность перестановки. Следовательно, каждое слагаемое суммы

det A = ∑a1,i1 a2,i2 Kan,in (−1)P{i1 ,i2 ,K,in } {i1 ,i2 ,K,in }

изменяет свой знак на противоположный.

4.Если матрица имеет нулевую строку (столбец), то определитель этой матрицы равен нулю:

26

Определители

L

L

L

L

L

 

ai,1

ai,2

ai,3

L ai,n

 

0

0

0

L

0

= 0

a j,1

a j,2

a j,3

L a j,n

 

L

L

L

L

L

 

Доказательство: Нуль является общим множителем строки (столбца) и, следовательно, общим множителем определителя.

5.Если две строки (столбца) матрицы равны между собой, то определитель этой матрицы равен нулю:

L L L L L

ai,1 ai,2 ai,3 L ai,n

L L L L L = 0

ai,1 ai,2 ai,3 L ai,n

L L L L L

Доказательство: По Свойству 3 при перестановке двух строк местами определитель изменяет свой знак. С другой стороны, перестановка местами одинаковых строк не меняет определитель. Следовательно,

det A = −det A det A = 0.

6.Если две строки (столбца) матрицы пропорциональны друг другу, то определитель этой матрицы равен нулю:

L

L

L

L

L

 

ai,1

ai,2

ai,3

L ai,n

 

L

L

L

L

L

= 0

cai,1 cai,2

cai,3

L cai,n

 

L

L

L

L

L

 

 

 

 

 

 

 

Доказательство: Вынося общий множитель пропорциональности в строке за знак определителя, мы получаем определитель, имеющий две одинаковых строки. Согласно Свойству 5 такой определитель равен нулю.

7.Определитель матрицы треугольного вида равен произведению элементов, стоящих на главной диагонали:

27

Определители

a1,1

a1,2

a1,3

L a1,n

 

0

a2,2

a2,3

L a2,n

 

0

0

a3,3

L a3,n

= a1,1 a2,2 a3,3 K an,n .

L

L

L

L

L

 

0

0

0

L

an,n

 

В частности, определитель единичной матрицы E равен 1.

Доказательство: По определению, det A представляет собой алгебраическую сумму произведений элементов (с учетом правила выбора знаков), составленных так, чтобы каждая строка и каждый столбец матрицы A были представлены в произведении одним и только одним элементом.

В первом столбце имеется только один ненулевой элемент, а именно

a1,1 .

Во втором столбце в нашем распоряжении остается только один ненулевой элемент a2,2 , поскольку первая строка уже представлена

своим элементом. При любой другом выборе соответствующее произведение обращается в нуль.

Аналогично, в третьем столбце мы можем остановить свой выбор только на элементе a3,3 и т.д.

Таким образом, сумма (2) содержит только одно ненулевое слагаемое, которое равно произведению элементов, стоящих на главной диагонали.

8.Если каждый элемент строки (столбца) определителя представлен в виде суммы двух членов, то

28

Определители

Доказательство: Преобразуем исходный определитель:

∑a1,i1 K(ak,ik +bk ,ik )Kan,in (−1)P{i1,i2 ,K,in} =

{i1,i2 ,K,in}

= ∑Kak,ik K + ∑Kbk,ik K=

{i1,i2 ,K,in} {i1,i2 ,K,in}

9.Определитель не изменится, если к одной из его строк прибавить другую, предварительно умноженную на любое число:

L

L

L

L

L

 

L

L

L L

L

ai,1

ai,2

ai,3

L ai,n

 

ai,1

ai,2

L L ai,n

L L L

L L

=

L

L L L L

ak ,1

ak ,2

ak ,3

L ak ,n

 

ak ,1 + cai,1

ak ,2 + cai,2

L L ak ,n + cai,n

L

L

L

L

L

 

L

L

L L

L

 

 

 

 

 

 

 

 

 

 

Доказательство: Определитель, стоящий в правой части равенства, можно представить в виде суммы двух определителей, один из которых является исходным, а второй имеет две пропорциональные друг другу строки и, следовательно, равен нулю.

10.Пусть A и B – квадратные матрицы одного и того же порядка. Тогда определитель произведения матриц равен произведению определителей:

det(AB) =det A det B .

29

Определители

 

 

 

 

 

 

 

 

Примеры.

 

 

 

 

 

 

 

 

1) Вычислить

 

sin x

cos x

 

.

 

 

 

−cos x

sin x

 

Решение:

 

 

 

 

 

 

 

 

 

sin x

cos x

 

=sin x sin x −cos x (−cos x) =sin2 x + cos2 x =1.

 

 

 

−cos x

sin x

 

 

 

 

 

2)Для данной матрицы A = a

c

тождества det A = det AT .

Решение:

 

 

 

det A =

 

a

b

 

= ad −bc

 

 

 

 

c

d

 

 

b убедиться в справедливости d

и

det AT =

 

a

c

 

= ad −bc.

 

 

 

b

d

 

3) Вычислить

1

2

3

 

4

5

6

.

 

7

8

9

 

Решение: Преобразуем определитель, вычитая из второй строки удвоенную первую, а из третьей – утроенную первую.

1

2

3

r2

→r2

−2r1

1

2

3

 

4

5

6

r3

→r3

−3r1

2

1

0

.

 

=

7

8

9

 

 

 

4

2

0

 

Вынося из третьей строки общий множитель 2, мы получаем определитель, имеющий две одинаковых строки:

1

2

3

 

1

2

3

 

2

1

0

= 2

2

1

0

=0 .

4

2

0

 

2

1

0

 

4) Пусть

5

0

 

7

1

 

A =

1

2

и

B =

3

2

. Убедиться в справедливости

 

 

 

 

 

тождества det AB =det A det B.

Решение:

30

Определители

 

 

det A =

 

5

0

 

=10 ,

det B =

 

 

7

1

 

=11,

 

 

 

 

 

 

 

 

 

 

 

 

 

 

1

2

 

 

 

 

 

 

 

 

 

 

3

2

 

 

 

 

 

 

 

 

 

 

 

det A det B =110 .

 

 

 

 

 

 

 

AB =

5 0 7 1

 

35 5

 

 

 

 

 

 

 

 

=

 

 

 

 

 

,

 

 

 

 

 

 

 

 

 

1 2 3 2

 

13 5

 

det AB =

 

35

5

 

=

5

 

35

1

 

=5(35 −13) =110 .

 

 

 

 

 

 

 

 

 

 

13

5

 

 

 

 

 

 

13

1

 

 

 

 

 

 

 

 

5)

Вычислить det A1000 , если

 

 

3

1

 

 

 

 

 

 

A =

2

 

.

 

 

 

 

 

 

 

 

 

 

 

 

 

 

 

 

 

 

 

 

 

 

 

 

1

 

 

 

 

 

 

Решение: Заметим, что

det A1000 =(det A)1000

 

 

 

Далее,

 

3

1

 

 

 

 

 

 

 

 

 

 

 

 

 

 

 

 

 

 

 

 

 

det A =

= 3 −2 =1

 

 

 

det A1000 =11000 =1.

 

2

1

 

 

 

2

3

 

4

 

 

 

 

 

 

 

 

 

 

 

 

 

6)

 

0

 

1

 

5

 

 

 

 

 

 

 

 

 

 

 

 

 

Пусть A =

 

 

. Вычислить:

 

 

 

 

 

 

 

 

0

 

0

 

 

 

 

 

 

 

 

 

 

 

 

 

 

 

 

 

 

 

 

 

−1

 

 

 

 

 

 

 

 

 

 

 

 

 

(a) det A,

 

 

 

 

 

(b) det A3 ,

 

 

 

(c) det(2A) ,

 

(d) det(−3A) ,

 

 

 

 

(e)

det(A − 2E) .

 

 

 

 

 

 

Решение:

(a)Определитель матрицы треугольного вида равен произведению диагональных элементов.

Таким образом,

2 3 4

det A = 0 1 5 = 2 1 (−1) = −2 .

00 −1

(b)Определитель произведения матриц равен произведению определителей. Следовательно,

det A3 = (det A)3 = (−2)3 = −8.

(c) Представим матрицу 2A в виде 2E A, где E– единичная матрица. Тогда

31

Вычисление определителя 2 го порядка. Определитель матрицы

Лекция 2. определители

    Определители второго порядка

    Определители третьего порядка

    Алгебраические дополнения и миноры

    Разложение определителя по строке или столбцу

    Свойства определителей

    Обратная матрица

    Свойства обратной матрицы

1. Определители второго порядка

Понятие определителя вводится только для квадратной матрицы .

Определитель – это число, которое считается по определенным правилам. Порядок определителя – это порядок квадратной матрицы. Если для задания матриц использовались круглые скобки, то в теории определителей используют прямые скобки.

Каждой квадратной матрице поставим в соответствие некоторое число, которое будем называть определителем матрицы, и укажем правило его вычисления. Обозначения:


.

Пример 1.
.

2. Определители третьего порядка


В каждом произведении нет чисел из одного столбца или одной строки.

Приведем схему для запоминания порядка получения слагаемых в определителе.

Произведение чисел на одной диагонали берется со знаком «+» (это главная диагональ матрицы), а на другой – с противоположным знаком.

Пример 2 .

3. Алгебраические дополнения и миноры

Для вычисления определителей порядка больше третьего применяют другие способы вычисления.

Пример 3. Минор
определителя есть.

.

Полезно запомнить, что
и
.

Пример 4. В примере 3алгебраическое дополнение

4. Разложение определителя по строке или столбцу

Вычисление определителя -го порядка можно свести к вычислению определителей порядка
, используя следующие формулы.

Это число равно сумме произведений элементов любой строки на их алгебраические дополнения .

Пример 5 . Вычислить определитель третьего порядка
разложением по первой строке.

Решение

Это число равно сумме произведений элементов любого -го столбца на их алгебраические дополнения.

Независимо от способа разложения всегда получается один и тот же ответ.

5. Свойства определителей

1. При транспонировании квадратной матрицы ее определитель не меняется:
.

Вывод. Свойства определителей, сформулированных для строк, справедливы и для столбцов.

2. При перестановке двух строк (столбцов) определитель меняет знак на противоположный. Например,
.

3. Определитель равен нулю , если:

а) он имеет нулевую строку (столбец)
;

б) он имеет пропорциональные (одинаковые) строки (столбец)
.

4. Общий множитель в строке (столбце) можно выносить за знак определителя. Например,
.

5. Определитель не изменяется , если к элементам какой-либо строки прибавить (вычесть) соответствующие элементы другой строки, умноженные на любое число.

Например,
.

6. Если в определителе каждый элемент строки есть сумма двух слагаемых, то этот определитель равен сумме двух определителей:

.

7. Определитель произведения двух квадратных матриц одного и того же порядка равен произведению определителей этих матриц:

.

8. Определитель квадратной матрицы треугольного вида равен произведению элементов, стоящих на главной диагонали:

.

6. Обратная матрица

Вместо операции деления матриц вводится понятие обратной матрицы.

Обозначается обратная матрица
, то есть .

Очевидна аналогия с числами: для числа 2 число ½ есть обратное, так как
. Именно поэтому матрица, обратная к А, обозначается
.

Теорема «Необходимое и достаточное условие существования обратной матрицы». Для того чтобы квадратная матрица имела обратную матрицу
, необходимо и достаточно, чтобы определитель матрицыбыл не равен нулю.

Правило нахождения обратной матрицы

0) Смотрим, является ли матрица квадратной. Если нет, то обратной матрицы не существует; если квадратная, то переходим к пункту 1.

1) Вычисляем определитель матрицы
: если он не равен нулю, то обратная матрица существует:
; если равен нулю, то обратной матрицы нет.

2) Для каждого элемента матрицы вычисляем его алгебраическое дополнение.

3) Составляем матрицу из алгебраических дополнений, которая затем транспонируем:
.

4) Каждый элемент матрицы
делим на определитель:
Получаем матрицу, обратную данной.

7. Нахождение обратной матрицы для матриц второго порядка

Пример 6. Дана матрица
. Найти обратную матрицу.

Решение .


Проверка. Убедимся, что найдена действительно обратная матрица. Найдем произведение матриц и
.

8. Свойства обратной матрицы

1.
,

где А и В – невырожденные квадратные матрицы одинакового порядка.

2.
.

3.
.

4.
.

Контрольные вопросы

    Что называется определителем второго порядка?

    Как вычислить определитель третьего порядка?

    Как вычислить определитель 3 порядка по правилу треугольников?

    Что называется алгебраическим дополнением элемента определителя? Приведите примеры для определителей 2 и 3 порядков.

    Напишите разложения определителя третьего порядка по элементам произвольной строки и произвольного столбца.



Определителем квадратной матрицы называется число, которое вычисляется следующим образом:

а) Если порядок квадратной матрицы равен 1, т.е. она состоит из 1 числа, то определитель равен этому числу;

б)Если порядок квадратной матрицы равен 2, т.е. она состоит из 4 чисел, то определитель равен разности произведения элементов главной диагонали и произведения элементов побочной диагонали;

в)Если порядок квадратной матрицы равен 3, т.е. она состоит из 9 чисел, то определитель равен сумме произведений элементов главной диагонали и двух треугольников параллельных этой диагонали, из которой вычли сумму произведений элементов побочной диагонали и двух треугольников параллельных этой диагонали.

Примеры

Свойства определителей

1. Определитель не изменится, если строки заменить столбцами, а столбцы – строками

  1. Определитель, имеющий 2 одинаковых ряда, равен нулю
  2. Общий множитель какого – либо ряда (строки или столбца) определителя можно вынести за знак определителя

4. При перестановке двух параллельных рядов определитель меняет знак на противоположный

5. Если элементы какого-либо ряда определителя представляют собой суммы двух слагаемых, то определитель может быть разложен на сумму двух соответствующих определителей

6. Определитель не изменится, если к элементам одного ряда прибавить соответствующие элементы параллельного ряда, умноженные на любое число

Минор элемента определителя и его алгебраическое дополнение

Минором элемента a IJ определителя n-го порядка называется определитель n-1 порядка, полученный из исходного с помощью вычеркивания i-той строки и j-того столбца

Алгебраическое дополнение элемента a IJ определителя – это его минор, умноженный на (-1) i+ j

Пример

Обратная матрица

Матрица называется невырожденной , если ее определитель не равен нулю, в противном случае, матрицу называют вырожденной

Матрица называется союзной , если она состоит из соответствующих алгебраических дополнений и транспонирована

Матрица называется обратной к данной матрице, если их произведение равно единичной матрице того же порядка, что и данная матрица

Теорема о существовании обратной матрицы

Любая невырожденная матрица имеет обратную, равную союзной матрице, деленной на определитель данной матрицы

Алгоритм нахождения обратной матрицы А

  1. Вычислить определитель
  1. Транспонировать матрицу
  1. Составить союзную матрицу, вычислить все алгебраические дополнения транспонированной матрицы
  1. Воспользоваться формулой:

Минором матрицы называется определитель, состоящий из элементов, находящихся на пересечении выделенных k строк и k столбцов данной матрицы размера mxn

Рангом матрицы называется наибольший порядок того минора матрицы, который отличен от нуля

Обозначение r(A), rangA

Ранг равен количеству ненулевых строк ступенчатой матрицы.

Пример

Системы линейных уравнений.

Системой линейных уравнений, содержащей m уравнений и n неизвестных, называется система вида

где числа a IJ — коэффициенты системы,числа b i — свободные члены

Матричная форма записи системы линейных уравнений

Решением системы называются n значений неизвестных c 1 , c 2 ,…, c n , при подстановке которых в систему все уравнения системы обращаются в верные равенства. Решение системы можно записать в виде вектор – столбца.

Система уравнений называется совместной , если она имеет хотя бы одно решение, и несовместной , если решений не имеет.

Теорема Кронекера – Капелли

Система ЛУ совместна тогда и только тогда, когда ранг основной матрицы равен рангу расширенной

Методы решения системы ЛУ

1. Метод Гаусса (расширенную матрицу с помощью элементарных преобразований свести к ступенчатой, а потом к канонической)

К элементарным преобразованиям относятся:

Перестановка строк (столбцов)

Прибавление к одной строке (столбцу) другой, умноженной на число, отличное от 0.

Составим расширенную матрицу:

Выберем ведущий элемент, стоящий в первом столбце и первой строке, элемент 1., назовем его ведущим. Строка, в которой находится ведущий элемент меняться не будет. Обнулим элементы под главной диагональю. Для этого прибавим ко второй строке первую, умноженную на (-2). Прибавим к третьей строке первую, умноженную на (-1), получим:

Поменяем вторую и третью строки местами. Мысленно вычеркиваем первый столбец и первую строку и продолжаем алгоритм для оставшейся матрицы. К третьей строке прибавляем 2-ю, умноженную на 5.

Привели расширенную матрицу к ступенчатому виду. Возвращаясь к уравнениям системы, начиная с последней строки и двигаясь вверх, поочередно определяем неизвестные.

2. Матричный метод (AX=B, A -1 AX=A -1 B, X=A -1 B; матрицу, обратную к основной матрице умножить на столбец свободных членов)

3. Метод Крамера.

Решение системы находится по формуле:

Где -определитель измененной основной матрицы, в которой i-й столбец изменен на столбец свободных членов, а — главный определитель, состоящий из коэффициентов при неизвестных.

Векторы.

Вектор – это направленный отрезок

Любой вектор задается длиной (модулем) и направлением.

Обозначение: или

где А – начало вектора, В – конец вектора, – длина вектора.

Классификация векторов

Нулевой вектор – это вектор, длина которого равна нулю

Единичный вектор – это вектор, длина которого равна единице

Равные векторы – это два вектора, у которых совпадают длина и направление

Противоположные векторы – это два вектора, у которых длины равны, а направления – противоположные

Коллинеарные векторы – это два вектора, которые лежат на одной прямой или на параллельных прямых

Сонаправленные векторы – это два коллинеарных вектора с одинаковым направлением

Противоположно направленные векторы– это два коллинеарных вектора с противоположным направлением

Компланарные векторы – это три вектора, которые лежат в одной плоскости или на параллельных плоскостях

Прямоугольная система координат на плоскости – это две взаимно перпендикулярные прямые с выбранным направлением и началом отсчета, при этом горизонтальная прямая называется осью абсцисс, а вертикальная – осью ординат

Каждой точке в прямоугольной системе координат поставим в соответствие два числа: абсциссу и ординату

Прямоугольная система координат в пространстве – это три взаимно перпендикулярные прямые с выбранным направлением и началом отсчета, при этом горизонтальная прямая, направленная на нас, называется осью абсцисс, горизонтальная прямая, направленная вправо от нас — осью ординат, а вертикальная прямая, направленная вверх – осью аппликат

Каждой точке в прямоугольной системе координат поставим в соответствие три числа: абсциссу, ординату и аппликату

На практике часто исследователю приходится иметь дело с неизвестными величинами, связанными между собой некоторыми заранее определенными зависимостями, которые могут быть выражены любыми формулами. Если при этом выполняется ряд условий:

  1. коэффициенты в формулах постоянные,
  2. неизвестные входят в формулы только в первой степени,
  3. отсутствуют произведения между самими неизвестными,

то тогда такие зависимости называют линейными.

Пример . В лаборатории 10 образцов имеют общий вес 280 г. Найти средний вес одного образца, если тара весит 15 г.

Решение . Для ответа на вопрос воспользуемся простым уравнением:

обозначив за x средний вес одного образца. Решением составленного уравнения будет 26,5 г.

Пример . В лаборатории 10 образцов, поступивших от 1 отдела, и 10 образцов, поступивших от 2-го отдела, имеют общий вес 280 г, а 5 образцов из первого набора и 2 образца из второго набора имеют общий вес 128 г. Найти средний вес образцов в каждом наборе.

Решение . Для ответа на вопрос составим два уравнения, обозначив за x — средний вес образца породы 1, а за y — средний вес образца породы 2,

10x+10y=280; 5x+2y=128,

решая которые совместно, получаем x=24 г; y=4 г .

В обоих рассмотренных примерах мы имели дело с линейными зависимостями: в первом случае – с линейным уравнением , а во втором – с линейной системой уравнений .

Заменим коэффициенты буквами и получим линейную систему уравнений:

Определение 1 . Матрицей будем называть любую прямоугольную таблицу, составленную из чисел a ij

Определение 2 . Элементы a ij из которых составлена матрица, называют элементами данной матрицы

Определение 3 . Определителем второго порядка или детерминантом , соответствующим матрице (1.2) назовем число D такое, что

Определитель обозначается буквами D или и записывается

Следует обратить внимание, что хотя определитель есть число, по определению 3, но до тех пор пока не найдено его значение в виде единственного числа ( по формуле 1.2 или еще каким-либо допустимым способом), он записывается в виде таблицы. Тогда можно сказать, например, о перестановке строк или столбцов в этой таблице. В таком случае следует говорить » определитель , соответствующий матрице». Но на практике обычно вторая часть этой фразы для простоты опускается и тогда остается только одно слово – определитель . Для того, чтобы различить что имеется в виду – сам определитель в виде таблицы или его найденное значение , во втором случае используют слово детерминант. Поэтому, если говорят, например, «количество строк в определителе…», то имеют в виду определитель , соответствующий матрице, но еще не вычисленный до единственного числа. А, если говорят детерминант, то имеют в виду, что данный определитель представлен единственным числом, вычисленным либо по формуле, либо еще каким-нибудь допустимым способом.

Пример . Дана система уравнений

Составить матрицу системы и вычислить определитель .

Решение . Из коэффициентов системы составим матрицу: и соответствующий ей детерминант

Выполним вычисления по формуле (2), получим

Определение 4 . Количество строк (или столбцов) в определителе называется порядком определителя

В примере был вычислен определитель второго порядка.

Определители обладают следующими свойствами.

Свойство 1 . Определитель не изменится, если его строки заменить столбцами и наоборот.

Покажем это. Пусть дан определитель второго порядка

Заменим строки столбцами и снова вычислим получившийся определитель

Сравнивая D с D * можно убедиться, что D = D * .

Определение 5 . Операция замены строк столбцами (или наоборот) в определителе называется транспонированием.

Свойство 2 . При перестановке двух строк или столбцов определитель меняет свой знак.

Поверку этого свойства проведем на примере, как и для свойства 1. Пусть дан определитель

Поменяем в нем местами столбцы и вычислим получившийся определитель .

Сравнивая результаты, убеждаемся, что определитель , действительно, поменял свой знак. Поменяем теперь местами строки и вновь убедимся в справедливости данного свойства.

Определители и правило Крамера. Определители 2-го и 3-го порядка. Правило Крамера. Миноры и алгебраические дополнения. Разложение определителя по строке или столбцу. Основные свойства определителей Метод элементарных преобразований.

2. ОПРЕДЕЛИТЕЛИ И ПРАВИЛО КРАМЕРА

2.1. Определители второго порядка

Понятие определителя возникло также в связи с задачей решения систем линейных уравнений. Определитель (или детерминант ) есть число, характеризующее квадратную матрицу A и обозначается обычно символами: detA , | A | или . Если матрица задана явно, в виде таблицы, то определитель обозначают, заключая таблицу в вертикальные линии.

Определитель матрицы второго порядка находится следующим образом :

(2.1)
Он равен произведению элементов главной диагонали матрицы минус произведение элементов второй диагонали .

Например,

Следует еще раз подчеркнуть, что матрица есть таблица чисел, тогда как определитель есть число, определяемое через элементы квадратной матрицы.

Рассмотрим теперь систему двух линейных уравнений с двумя неизвестными:

Используя понятие определителя 2-го порядка, решение этой системы можно записать в виде:

(2.2)

Это есть правило Крамера решения системы двух линейных уравнений с двумя неизвестными при условии, что 0.

Пример 2.1. Решить систему линейных уравнений, используя правило Крамера:

Решение . Найдем определители:

Историческая справка. Идея понятия «определителя» могла бы принадлежать Г. Лейбницу (1646-1716), если бы он развил и опубликовал свои идеи относительно определителей, к которым он пришел в 1693 г. Поэтому приоритет в разработке метода определителей решения систем линейных уравнений принадлежит Г. Крамеру (1704-1752), который опубликовал свои исследования по этой теме в 1750 г. Однако Крамер не построил полноценной теории определителей, к тому же ему не доставало удобного обозначения. Первое обширное исследование , посвященное определителям, было А. Вандермондом (1735-1796) в 1772 г. Он дал логическое изложение теории определителей и ввел правило разложения определителя с помощью миноров. Полное изложение теории определителей было дано лишь в 1812 г.
Ж. Бине (1786-1856) и О. Коши (1789-1858). Термин «определитель» («детерминант» ) в современном его значении был введен Коши (ранее этот термин использовался К. Гауссом для обозначения дискриминанта квадратичной формы).

2.2. Определители третьего порядка

Определитель матрицы 3-го порядка находится следующим образом

(2.3)

Естественно, что запомнить эту формулу довольно трудно. Однако есть правила, которые облегчают выписывание выражения для определителя 3-го порядка.


Правило треугольников : три слагаемых, входящих в исходное выражение со знаком плюс, есть произведения элементов главной диагонали или треугольников, основания которых параллельны этой диагонали. Остальные три слагаемых, входящих со знаком минус, находятся таким же образом, но относительно второй диагонали.


Правило Саррюса : припишем к матрице справа первый, а затем второй столбец. Тогда «положительные» слагаемые будут находиться на линиях параллельных главной диагонали, а «отрицательные» на линиях, параллельных второй диагонали .

2.3. Правило Крамера

Рассмотрим систему 3-х уравнений с тремя неизвестными

Используя определители 3-го порядка, решение такой системы можно записать в таком же виде, как и для системы двух уравнений, т.е.

(2.4)

если 0. Здесь

Это есть правило Крамера решения системы трех линейных уравнений с тремя неизвестными .

Пример 2.3. Решить систему линейных уравнений при помощи правила Крамера:

Решение . Находим определитель основной матрицы системы

Поскольку 0, то для нахождения решения системы можно применить правило Крамера, но предварительно вычислим еще три определителя:

Проверка:

Следовательно, решение найдено правильно.

Правила Крамера, полученные для линейных систем 2-го и 3-го порядка, наводят на мысль, что такие же правила можно сформулировать и для линейных систем любого порядка. Действительно имеет место

Теорема Крамера. Квадратная система линейных уравнений с отличным от нуля определителем основной матрицы системы (0) имеет одно и только одно решение и это решение вычисляется по формулам

(2.5)

где  – определитель основной матрицы ,  i определитель матрицы , полученной из основной, заменой i -го столбца столбцом свободных членов .

Отметим, что если =0, то правило Крамера не применимо. Это означает, что система либо не имеет вообще решений, либо имеет бесконечно много решений.

Сформулировав теорему Крамера, естественно возникает вопрос о вычислении определителей высших порядков.

2.4. Определители n-го порядка

Дополнительным минором M ij элемента a ij называется определитель, получаемый из данного путем вычеркивания i -й строки и j -го столбца. Алгебраическим дополнением A ij элемента a ij называется минор этого элемента, взятого со знаком (–1) i + j , т.е. A ij = (–1) i + j M ij .

Например, найдем миноры и алгебраические дополнения элементов a 23 и a 31 определителя

Получаем

Используя понятие алгебраического дополнения можно сформулировать теорему о разложении определителя n -го порядка по строке или столбцу .

Теорема 2.1. Определитель матрицы A равен сумме произведений всех элементов некоторой строки (или столбца) на их алгебраические дополнения:

(2.6)

Данная теорема лежит в основе одного из основных методов вычисления определителей, т.н. метода понижения порядка . В результате разложения определителя n -го порядка по какой-либо строке или столбцу, получается n определителей (n –1)-го порядка. Чтобы таких определителей было меньше, целесообразно выбирать ту строку или столбец, в которой больше всего нулей. На практике формулу разложения определителя обычно записывают в виде:

т.е. алгебраические дополнения записывают в явном виде через миноры.

Примеры 2.4. Вычислить определители, предварительно разложив их по какой-либо строке или столбцу. Обычно в таких случаях выбирают такой столбец или строку, в которой больше всего нулей. Выбранную строку или столбец будем обозначать стрелкой.

2.5. Основные свойства определителей

Разлагая определитель по какой-либо строке или столбцу, мы получим n определителей (n –1)-го порядка. Затем каждый из этих определителей (n –1)-го порядка также можно разложить в сумму определителей (n –2)-го порядка. Продолжая этот процесс , можно дойти до определителей 1-го порядка, т.е. до элементов матрицы, определитель которой вычисляется. Так, для вычисления определителей 2-го порядка придется вычислить сумму двух слагаемых, для определителей 3-го порядка – сумму 6 слагаемых, для определителей 4-го порядка – 24 слагаемых. Число слагаемых будет резко возрастать по мере увеличения порядка определителя. Это означает, что вычисление определителей очень высоких порядков становится довольно трудоемкой задачей, непосильной даже для ЭВМ. Однако вычислять определители можно и по-другому, используя свойства определителей.

Свойство 1 . Определитель не изменится, если в нем поменять местами строки и столбцы, т.е. при транспонировании матрицы:

.

Данное свойство свидетельствует о равноправии строк и столбцов определителя. Иначе говоря, любое утверждение о столбцах определителя справедливо и для его строк и наоборот.

Свойство 2 . Определитель меняет знак при перестановке двух строк (столбцов).

Следствие . Если определитель имеет две одинаковые строки (столбца), то он равен нулю.

Свойство 3 . Общий множитель всех элементов в какой-либо строке (столбце) можно вынести за знак определителя.

Например,

Следствие . Если все элементы некоторой строки (столбца) определителя равны нулю, то и сам определитель равен нулю.

Свойство 4 . Определитель не изменится, если к элементам одной строки (столбца), прибавить элементы другой строки (столбца), умноженной на какое-либо число.

Например,

Свойство 5 . Определитель произведения матриц равен произведению определителей матриц:

2.6.

Теорема 2.2. Определитель треугольной матрицы равен произведению элементов главной диагонали:

Элементарными преобразованиями матрицы называются следующие преобразования: 1) умножение строки (столбца) на число, не равное нулю; 2) прибавление одной строки (столбца) к другой; 3) перестановка двух строк (столбцов).

Метод элементарных преобразований заключается в том, чтобы при помощи элементарных преобразований, учитывая свойства определителей, привести матрицу к треугольному виду.

Пример 2.5. Вычислить определитель при помощи элементарных преобразований, приведя их к треугольному виду:

Пример 2.6. Вычислить определитель:

.

Решение . Упростим данный определитель , а затем вычислим его:

. 
Пример 2.7. Вычислить определитель
.

Решение . Способ 1 .При помощи элементарных преобразований матрицы, учитывая свойства определителей, будем получать в какой-либо строке или столбце нули, а затем будем разлагать полученный определитель по этой строке или столбцу:


–6

2

-2

.
Способ 2 .При помощи элементарных преобразований матрицы, учитывая свойства определителей, приведем матрицу к треугольному виду:

. 

Вычисление определителей при помощи элементарных преобразований, путем приведения его к треугольному виду, является одним из самых распространенных методов. Это связано с тем, что он является основным методом при реализации вычислений определителей на ЭВМ. Точнее он является одной из модификаций метода Гаусса , который обычно используется при решении систем линейных уравнений.

Пример 2.8. Вычислить определитель методом Гаусса:

Решение. Рассмотрим первый столбец и выберем в нем ту строку, которая содержит 1. Если единиц нет, то нужно эту единицу создать при помощи элементарных преобразований: переставляя строки или столбцы, складывая или вычитая их друг с другом, умножая или деля их на какое-либо число (учитывая при этом, конечно свойства определителей). Возьмем за основу вторую строку и получим при помощи ее нули в первом столбце:

После этого на первую строку больше внимания не обращаем. Рассмотрим 2-й столбец.

В результате, получилась треугольная матрица. Для того чтобы вычислить определитель, осталось только перемножить элементы матрицы, находящиеся на главной диагонали. Таким образом, получаем ответ: –2(–1)(–1)1334 = –264. 

Нелинейные операции над матрицами.

Св-ва операций.

Заглавная страница
Избранные статьи
Случайная статья
Познавательные статьи
Новые добавления
Обратная связь

КАТЕГОРИИ:

Археология
Биология
Генетика
География
Информатика
История
Логика
Маркетинг
Математика
Менеджмент
Механика
Педагогика
Религия
Социология
Технологии
Физика
Философия
Финансы
Химия
Экология

ТОП 10 на сайте

Приготовление дезинфицирующих растворов различной концентрации

Техника нижней прямой подачи мяча.

Франко-прусская война (причины и последствия)

Организация работы процедурного кабинета

Смысловое и механическое запоминание, их место и роль в усвоении знаний

Коммуникативные барьеры и пути их преодоления

Обработка изделий медицинского назначения многократного применения

Образцы текста публицистического стиля

Четыре типа изменения баланса

Задачи с ответами для Всероссийской олимпиады по праву



Мы поможем в написании ваших работ!

ЗНАЕТЕ ЛИ ВЫ?

Влияние общества на человека

Приготовление дезинфицирующих растворов различной концентрации

Практические работы по географии для 6 класса

Организация работы процедурного кабинета

Изменения в неживой природе осенью

Уборка процедурного кабинета

Сольфеджио. Все правила по сольфеджио

Балочные системы. Определение реакций опор и моментов защемления

⇐ ПредыдущаяСтр 2 из 9Следующая ⇒

Умножение матриц (AB) есть оп-ция вычисления матрицы C, эл-ты кот. равны сумме произведений эл-ов в соответствующей строке 1ого множителя и столбце 2ого.

Правило: чтобы получить эл-т, стоящий в i -й строке и j -ом столбце прозвед-ия 2х матриц, нужно эл-ты i -й строки 1ой мат-цы умножить на соответствующие эл-ты j -го столбца 2ой мат-ры и получ. произвед-ия сложить.

В 1ом множителе должно быть столько же столбцов, сколько строк во 2ом. Если матрица A имеет размерность m×n, B — n×k, то размерность их произведения AB=C есть m×k.

Св — ва мат — ц : 1) A*(B*C)=(A*B)*C; 2)A*(B+C)=AB+AC; 3)(A+B)*C=CA+CB; 4)α*(A*B)=(αA)*B

Возводить в степень можно только квадрат. матрицы.

Транспонирование матрицы (АТ) – оп-ция, при кот. матрица отражается относительно главной диагонали.

Матрицу В называют тр анспонированнойматрицей А, а переход от А к В транспонированием мат-цы, если эл-ты каждой строки матрицы А записать в том же порядке в столбцы матрицы В. Обозначается АТ.

Другими словами, aij = bji.

Св-ва: 1) ; 2)

Определители. Основные понятия. Свойства определителей.

Квадр. матрице А порядка N можно сопоставить число det A (или |A|), называемое ее определителем следующим образом:

Определитель мат-цы А также наз. ее детерминантом. Методы, позволяющие реализовать вычисление определ-ей высоких порядков на основе определителей низших порядков.


Один из методов основан на св-ве разложения определителя по элементам некоторого ряда. При этом заметим, что определ-ли невысоких порядков (1, 2, 3) желательно уметь вычислять согласно определению.

Вычисление определ-ля 2-го порядка иллюстрируется схемой:

При вычислении определителя 3-го порядка удобно пользоваться правилом треугольников (или Саррюса):

Свойства определителей

1. Определитель не изменится, если его строки заменить столбцами, и наоборот.

2. При перестановке двух параллельных рядов определитель меняет знак

3. Общий множитель элементов какого-либо ряда определителя можно вынести за знак определителя.

4. Определитель, имеющий два одинаковых ряда, равен нулю.

Из св-в 3 и 4 следует, что если все эл-ты некотор. ряда пропорциональны соответствующим эл-там параллельного ряда, то такой определитель равен нулю.

5.Вычисление определителей 2-го и 3-го порядков. Св-ва определит.(сумма опред., тождественные преобраз., сумма произвед. эл-ов строк и столбцов)

Вычисление определ-ля 2-го порядка иллюстрируется схемой:

При вычислении определителя 3-го порядка удобно пользоваться правилом треугольников (или Саррюса):

Свойства определителей

1. Если эл-ты какого-либо ряда опред-ля представляют собой суммы 2х слагаемых, то опред-тель может быть разложен на сумму 2х соответствующих опред-лей.

2. Определитель не изменится, если к эл-там 1ого ряда прибавить соответствующие эл-ты параллельного ряда, умноженные на любое число.

3. Определитель произвед-ия 2х квадр. матр-иц : C=A*B равен произвед-ию detС= detВ* detА.

4.Сумма произвед-ий эл-ов строки или столбца,по кот. раскрывается опрделит. умноженный на алгебраич. дополнение параллельного столбца или строки равен нулю.

⇐ Предыдущая123456789Следующая ⇒



Читайте также:



Техника прыжка в длину с разбега

Организация работы процедурного кабинета

Области применения синхронных машин

Оптимизация по Винеру и Калману



Последнее изменение этой страницы: 2020-10-24; просмотров: 134; Нарушение авторского права страницы; Мы поможем в написании вашей работы!

infopedia. su Все материалы представленные на сайте исключительно с целью ознакомления читателями и не преследуют коммерческих целей или нарушение авторских прав. Обратная связь — 176.9.44.166 (0.006 с.)

Математические матрицы. Основные операции над матрицами

Матрицы, познакомьтесь с ее основными понятиями. Определяющими элементами матрицы являются ее диагонали — и побочная. Главная начинается с элемента в первом ряду, первом столбце и продолжается до элемента последнего столбца, последнего ряда (то есть идет слева направо). Побочная же диагональ начинается наоборот в первом ряду, но последнем столбце и продолжается до элемента, имеющего координаты первого столбца и последнего ряда (идет справа налево).

Для того чтобы перейти к следующим определениям и алгебраическим операциям с матрицами, изучите виды матриц. Самые простые из них — это квадратная, единичная, нулевая и обратная. В совпадает число столбцов и строк. Транспонированная матрица, назовем ее В, получается из матрицы А, путем замены столбцов на строки. В единичной все элементы главной диагонали — единицы, а другие — нули. А в нулевой даже элементы диагоналей нулевые. Обратная матрица — это та, на которую исходная матрица приходит к единичному виду.

Также матрица может быть симметрична относительно главной или побочной осей. То есть элемент, имеющий координаты а(1;2), где 1 — это номер строки, а 2 — столбца, равен а(2;1). А(3;1)=А(1;3) и так далее. Матрицы согласованными — это те, где количество столбцов одной равно количеству строк другой (такие матрицы можно перемножать).

Главные действия, которые можно совершить с матрицами — это сложение, умножение и нахождение определителя. Если матрицы одинакового размера, то есть имеют равное количество строк и столбцов, то их можно сложить. Складывать необходимо элементы, стоящие на одинаковых местах в матрицах, то есть а (m;n) сложите с в (m;n), где m и n — это соответствующие координаты столбца и строки. При сложении матриц действует главное правило обычного арифметического сложения — при перемене мест слагаемых сумма не меняется. Таким образом, если вместо простого элемента а стоит выражение а+в, то его можно сложить в элементом с другой соразмерной матрицы по правилам а+(в+с)= (а+в)+с.

Умножать можно согласованные матрицы, которым дано выше. При этом получается матрица, где каждый элемент — это сумма попарно перемноженных элементов строки матрицы А и столбца матрицы В. При перемножении очень важен порядок действий. m*n не равно n*m.

Также одно из главных действий — это нахождение . Еще его называют детерминантом и обозначают так: det. Эта величина определяется по модулю, то есть никогда не бывает отрицательной. Легче всего найти детерминант у квадратной матрицы 2х2. Для этого необходимо перемножить элементы главной диагонали и вычесть из них перемноженные элементы побочной диагонали.

Данное методическое пособие поможет Вам научиться выполнять действия с матрицами : сложение (вычитание) матриц, транспонирование матрицы, умножение матриц, нахождение обратной матрицы. Весь материал изложен в простой и доступной форме, приведены соответствующие примеры, таким образом, даже неподготовленный человек сможет научиться выполнять действия с матрицами. Для самоконтроля и самопроверки Вы можете бесплатно скачать матричный калькулятор >>> .

Я буду стараться минимизировать теоретические выкладки, кое-где возможны объяснения «на пальцах» и использование ненаучных терминов. Любители основательной теории, пожалуйста, не занимайтесь критикой, наша задача – научиться выполнять действия с матрицами .

Для СВЕРХБЫСТРОЙ подготовки по теме (у кого «горит») есть интенсивный pdf-курс Матрица, определитель и зачёт!

Матрица – это прямоугольная таблица каких-либо элементов . В качестве элементов мы будем рассматривать числа, то есть числовые матрицы. ЭЛЕМЕНТ – это термин. Термин желательно запомнить, он будет часто встречаться, не случайно я использовал для его выделения жирный шрифт.

Обозначение: матрицы обычно обозначают прописными латинскими буквами

Пример: рассмотрим матрицу «два на три»:

Данная матрица состоит из шести элементов :

Все числа (элементы) внутри матрицы существуют сами по себе, то есть ни о каком вычитании речи не идет:

Это просто таблица (набор) чисел!

Также договоримся не переставлять числа, если иного не сказано в объяснениях. У каждого числа свое местоположение, и перетасовывать их нельзя!

Рассматриваемая матрица имеет две строки:

и три столбца:

СТАНДАРТ : когда говорят о размерах матрицы, то сначала указывают количество строк, а только потом – количество столбцов. Мы только что разобрали по косточкам матрицу «два на три».

Если количество строк и столбцов матрицы совпадает, то матрицу называют квадратной , например: – матрица «три на три».

Если в матрице один столбец или одна строка , то такие матрицы также называют векторами .

На самом деле понятие матрицы мы знаем еще со школы, рассмотрим, например точку с координатами «икс» и «игрек»: . По существу, координаты точки записаны в матрицу «один на два». Кстати, вот Вам и пример, почему порядок чисел имеет значение: и – это две совершенно разные точки плоскости.

Теперь переходим непосредственно к изучению действий с матрицами :

1) Действие первое. Вынесение минуса из матрицы (внесение минуса в матрицу) .

Вернемся к нашей матрице . Как вы наверняка заметили, в данной матрице слишком много отрицательных чисел. Это очень неудобно с точки зрения выполнения различных действий с матрицей, неудобно писать столько минусов, да и просто в оформлении некрасиво выглядит.

Вынесем минус за пределы матрицы, сменив у КАЖДОГО элемента матрицы знак :

У нуля, как Вы понимаете, знак не меняется, ноль – он и в Африке ноль.

Обратный пример: . Выглядит безобразно.

Внесем минус в матрицу, сменив у КАЖДОГО элемента матрицы знак :

Ну вот, гораздо симпатичнее получилось. И, самое главное, выполнять какие-либо действия с матрицей будет ПРОЩЕ. Потому что есть такая математическая народная примета: чем больше минусов – тем больше путаницы и ошибок .

2) Действие второе. Умножение матрицы на число .

Пример:

Всё просто, для того чтобы умножить матрицу на число, нужно каждый элемент матрицы умножить на данное число. В данном случае – на тройку.

Еще один полезный пример:

– умножение матрицы на дробь

Сначала рассмотрим то, чего делать НЕ НАДО :

Вносить дробь в матрицу НЕ НУЖНО, во-первых, это только затрудняет дальнейшие действия с матрицей, во-вторых, затрудняет проверку решения преподавателем (особенно, если – окончательный ответ задания).

И, тем более, НЕ НАДО делить каждый элемент матрицы на минус семь:

Из статьи Математика для чайников или с чего начать , мы помним, что десятичных дробей с запятой в высшей математике стараются всячески избегать.

Единственное, что желательно сделать в этом примере – это внести минус в матрицу:

А вот если бы ВСЕ элементы матрицы делились на 7 без остатка , то тогда можно (и нужно!) было бы поделить.

Пример:

В этом случае можно и НУЖНО умножить все элементы матрицы на , так как все числа матрицы делятся на 2 без остатка .

Примечание: в теории высшей математики школьного понятия «деление» нет. Вместо фразы «это поделить на это» всегда можно сказать «это умножить на дробь». То есть, деление – это частный случай умножения.

3) Действие третье. Транспонирование матрицы .

Для того чтобы транспонировать матрицу, нужно ее строки записать в столбцы транспонированной матрицы.

Пример:

Транспонировать матрицу

Строка здесь всего одна и, согласно правилу, её нужно записать в столбец:

– транспонированная матрица.

Транспонированная матрица обычно обозначается надстрочным индексом или штрихом справа вверху.

Пошаговый пример:

Транспонировать матрицу

Сначала переписываем первую строку в первый столбец:

Потом переписываем вторую строку во второй столбец:

И, наконец, переписываем третью строку в третий столбец:

Готово. Грубо говоря, транспонировать – это значит повернуть матрицу набок.

4) Действие четвертое. Сумма (разность) матриц .

Сумма матриц действие несложное.
НЕ ВСЕ МАТРИЦЫ МОЖНО СКЛАДЫВАТЬ. Для выполнения сложения (вычитания) матриц, необходимо, чтобы они были ОДИНАКОВЫМИ ПО РАЗМЕРУ.

Например, если дана матрица «два на два», то ее можно складывать только с матрицей «два на два» и никакой другой!

Пример:

Сложить матрицы и

Для того чтобы сложить матрицы, необходимо сложить их соответствующие элементы :

Для разности матриц правило аналогичное, необходимо найти разность соответствующих элементов .

Пример:

Найти разность матриц ,

А как решить данный пример проще, чтобы не запутаться? Целесообразно избавиться от лишних минусов, для этого внесем минус в матрицу :

Примечание: в теории высшей математики школьного понятия «вычитание» нет. Вместо фразы «из этого вычесть это» всегда можно сказать «к этому прибавить отрицательное число». То есть, вычитание – это частный случай сложения.

5) Действие пятое. Умножение матриц .

Какие матрицы можно умножать?

Чтобы матрицу можно было умножить на матрицу нужно, чтобы число столбцов матрицы равнялось числу строк матрицы .

Пример:
Можно ли умножить матрицу на матрицу ?

Значит, умножать данные матрицы можно.

А вот если матрицы переставить местами, то, в данном случае, умножение уже невозможно!

Следовательно, выполнить умножение невозможно:

Не так уж редко встречаются задания с подвохом, когда студенту предлагается умножить матрицы, умножение которых заведомо невозможно.

Следует отметить, что в ряде случаев можно умножать матрицы и так, и так.
Например, для матриц, и возможно как умножение , так и умножение

Сложение матриц $ A $ и $ B $ это арифметическая операция, в результате которой, должна получаться матрица $ C $, каждый элемент которой равен сумме соответствующих элементов складываемых матриц:

$$ c_{ij} = a_{ij} + b_{ij} $$

Более подробно формула сложения двух матриц выглядит так:

$$ A + B = \begin{pmatrix} a_{11} & a_{12} & a_{13} \\ a_{21} & a_{22} & a_{23} \\ a_{31} & a_{32} & a_{33} \end{pmatrix} + \begin{pmatrix} b_{11} & b_{12} & b_{13} \\ b_{21} & b_{22} & b_{23} \\ b_{31} & b_{32} & b_{33} \end{pmatrix} = $$

$$ = \begin{pmatrix} a_{11} + b_{11} & a_{12}+b_{12} & a_{13}+b_{13} \\ a_{21}+b_{21} & a_{22}+b_{22} & a_{23}+b_{23} \\ a_{31}+b_{31} & a_{32}+b_{32} & a_{33}+b_{33} \end{pmatrix} = C $$

Обратите внимание, что складывать и вычитать матрицы можно только одинаковой размерности. При сумме или разности будет получаться матрица $ C $ такой же размерности как и слагаемые (вычитаемые) матрицы $ A $ и $ B $. Если матрицы $ A $ и $ B $ отличаются друг от друга размерами, то сложение (вычитание) таких матриц будет ошибкой!

В формуле складываются матрицы 3 на 3, значит и получиться должна матрица 3 на 3.

Вычитание матриц полностью аналогично по алгоритму сложения, только знак минус. Каждый элемент искомой матрицы $ C $ получается благодаря вычитанию соответствующих элементов матриц $ A $ и $ B $:

$$ c_{ij} = a_{ij} — b_{ij} $$

Запишем подробную формулу вычитания двух матриц:

$$ A — B = \begin{pmatrix} a_{11} & a_{12} & a_{13} \\ a_{21} & a_{22} & a_{23} \\ a_{31} & a_{32} & a_{33} \end{pmatrix} — \begin{pmatrix} b_{11} & b_{12} & b_{13} \\ b_{21} & b_{22} & b_{23} \\ b_{31} & b_{32} & b_{33} \end{pmatrix} = $$

$$ = \begin{pmatrix} a_{11} — b_{11} & a_{12}-b_{12} & a_{13}-b_{13} \\ a_{21}-b_{21} & a_{22}-b_{22} & a_{23}-b_{23} \\ a_{31}-b_{31} & a_{32}-b_{32} & a_{33}-b_{33} \end{pmatrix} = C $$

Стоит так же заметить, что нельзя складывать и вычитать матрицы с обычными числами, а так же с другими какими-то элементами

Будет полезно знать для дальнейших решений задач с матрицами знать свойства сложения (вычитания).

Свойства

  1. Если матрицы $ A,B,C $ одинаковые по размеру, тогда для них действует свойство ассоциативности: $$ A + (B + C) = (A + B) + C $$
  2. Для каждой матрицы существует нулевая матрица, обозначаемая $ O $, при сложении (вычитании) с которой исходная матрица не изменяется: $$ A \pm O = A $$
  3. Для каждой ненулевой матрицы $ A $ есть противоположная матрица $ (-A) $ сумма с которой обращается в нуль: $$ A + (-A) = 0 $$
  4. При сложении (вычитании) матриц допустимо свойство коммутативности, то есть матрицы $ A $ и $ B $ можно менять местами: $$ A + B = B + A $$ $$ A — B = B — A $$

Примеры решений

Пример 1

Даны матрицы $ A = \begin{pmatrix} 2&3 \\ -1& 4 \end{pmatrix} $ и $ B = \begin{pmatrix} 1&-3 \\ 2&5 \end{pmatrix} $.

Выполнить сложение матриц, а затем вычитание.

Решение

Первым делом проверяем матрицы на размерность. У матрицы $ A $ размерность $ 2 \times 2 $, у второй матрицы $ B $ размерность тоже $ 2 \times 2 $. Это значит, что с данными матрицами можно провести совместную операцию по сложению и вычитанию.

Напомним, что для суммы нужно выполнить попарное сложение соответствующих элементов матриц $ A \text{ и } B $.

$$ A + B = \begin{pmatrix} 2&3 \\ -1& 4 \end{pmatrix} + \begin{pmatrix} 1&-3 \\ 2&5 \end{pmatrix} = $$

$$ = \begin{pmatrix} 2 + 1 & 3 + (-3) \\ -1 + 2 & 4 + 5 \end{pmatrix} = \begin{pmatrix} 3 & 0 \\ 1 & 9 \end{pmatrix} $$

Аналогично сумме находим разность матриц с помощью замены знака «плюс» на «минус»:

$$ A — B = \begin{pmatrix} 2&3 \\ -1& 4 \end{pmatrix} + \begin{pmatrix} 1&-3 \\ 2&5 \end{pmatrix} = $$

$$ = \begin{pmatrix} 2 — 1 & 3 — (-3) \\ -1 — 2 & 4 — 5 \end{pmatrix} = \begin{pmatrix} 1 & 6 \\ -3 & -1 \end{pmatrix} $$

Если не получается решить свою задачу, то присылайте её к нам. Мы предоставим подробное решение. Вы сможете ознакомиться с ходом вычисления и почерпнуть информацию. Это поможет своевременно получить зачёт у преподавателя!

Ответ

$$ A + B = \begin{pmatrix} 3 & 0 \\ 1 & 9 \end{pmatrix}; A — B = \begin{pmatrix} 1 & 6 \\ -3 & -1 \end{pmatrix} $$

В статье: «Сложение и вычитание матриц» были даны определения, правила, замечания, свойства операций и практические примеры решения.

Матрицей размерности называется прямоугольная таблица, состоящая изэлементов, расположенных вm строках и n столбцах.

Элементы матрицы (первый индексi − номер строки, второй индекс j − номер столбца) могут быть числами, функциями и т. п. Матрицы обозначают заглавными буквами латинского алфавита.

Матрица называется квадратной , если у нее число строк равно числу столбцов (m = n ). В этом случае число n называется порядком матрицы, а сама матрица называется матрицей n -го порядка.

Элементы с одинаковыми индексами образуютглавную диагональ квадратной матрицы, а элементы (т.е. имеющие сумму индексов, равнуюn +1) − побочную диагональ .

Единичной матрицей называется квадратная матрица, все элементы главной диагонали которой равны 1, а остальные элементы равны 0. Она обозначается буквой Е .

Нулевая матрица − это матрица, все элементы которой равны 0. Нулевая матрица может быть любого размера.

К числу линейных операций над матрицами относятся:

1) сложение матриц;

2) умножение матриц на число.

Операция сложения матриц определена только для матриц одинаковой размерности.

Суммой двух матриц А и В называется матрица С , все элементы которой равны суммам соответствующих элементов матриц А и В :

.

Произведением матрицы А на число k называется матрица В , все элементы которой равны соответствующим элементам данной матрицы А , умноженным на число k :

Операция умножения матриц вводится для матриц, удовлетворяющих условию: число столбцов первой матрицы равно числу строк второй.

Произведением матрицы А размерности на матрицу В размерности называется матрицаС размерности , элементi -ой строки и j -го столбца которой равен сумме произведений элементов i -ой строки матрицы А на соответствующие элементы j -го столбца матрицы В :

Произведение матриц (в отличие от произведения действительных чисел) не подчиняется переместительному закону, т.е. в общем случае А В В А .

1.2. Определители. Свойства определителей

Понятие определителя вводится только для квадратных матриц.

Определителем матрицы 2-го порядка называется число, вычисляемое по следующему правилу

.

Определителем матрицы 3-го порядка называется число, вычисляемое по следующему правилу:

Первое из слагаемых со знаком «+» представляет собой произведение элементов, расположенных на главной диагонали матрицы (). Остальные два содержат элементы, расположенные в вершинах треугольников с основанием, параллельным главной диагонали (и). Со знаком «-» входят произведения элементов побочной диагонали () и элементов, образующих треугольники с основаниями, параллельными этой диагонали (и).

Это правило вычисления определителя 3-го порядка называется правилом треугольников (или правилом Саррюса).

Свойства определителей рассмотрим на примере определителей 3-го порядка.

1. При замене всех строк определителя на столбцы с теми же номерами, что и строки, определитель своего значения не меняет, т.е. строки и столбцы определителя равноправны

.

2. При перестановке двух строк (столбцов) определитель меняет свой знак.

3. Если все элементы некоторой строки (столбца) нули, то определитель равен 0.

4. Общий множитель всех элементов строки (столбца) можно вынести за знак определителя.

5. Определитель, содержащий две одинаковые строки (столбца), равен 0.

6. Определитель, содержащий две пропорциональные строки (столбца), равен нулю.

7. Если каждый элемент некоторого столбца (строки) определителя представляет сумму двух слагаемых, то определитель равен сумме двух определителей, в одном из которых в том же столбце (строке) стоят первые слагаемые, а в другом − вторые. Остальные элементы у обоих определителей одинаковые. Так,

.

8. Определитель не изменится, если к элементам какого-либо его столбца (строки) прибавить соответствующие элементы другого столбца (строки), умноженные на одно и то же число.

Это понятие, которое обобщает все возможные операции, производимые с матрицами. Математическая матрица — таблица элементов. О такой таблице, где m строк и n столбцов, говорят, что это матрица имеет размерность m на n .

Общий вид матрицы:

Для решения матриц необходимо понимать, что такое матрица и знать основные ее параметры. Основные элементы матрицы:

  • Главная диагональ, состоящая из элементов а 11 ,а 22 …..а mn .
  • Побочная диагональ, состоящая из элементов а 1n ,а 2n-1 …. .а m1 .

Основные виды матриц:

  • Квадратная — такая матрица, где число строк = числу столбцов (m=n ).
  • Нулевая — где все элементы матрицы = 0.
  • Транспонированная матрица — матрица В , которая была получена из исходной матрицы A путем замены строк на столбцы.
  • Единичная — все элементы главной диагонали = 1, все остальные = 0.
  • Обратная матрица — матрица, при умножении на которую исходная матрица даёт в результате единичную матрицу.

Матрица может быть симметричной относительно главной и побочной диагонали. Т.е., если а 12 =а 21 , а 13 =а 31 ,….а 23 =а 32 …. а m-1n =а mn-1 , то матрица симметрична относительно главной диагонали. Симметричными могут быть лишь квадратные матрицы.

Методы решения матриц.

Почти все методы решения матрицы заключаются в нахождении ее определителя n -го порядка и большинство из них довольно громоздки. Чтобы найти определитель 2го и 3го порядка есть другие, более рациональные способы.

Нахождение определителей 2-го порядка.

Для вычисления определителя матрицы А 2го порядка, необходимо из произведения элементов главной диагонали вычесть произведение элементов побочной диагонали:

Методы нахождения определителей 3го порядка.

Ниже приведены правила для нахождения определителя 3го порядка.

Упрощенно правило треугольника, как одного из методов решения матриц , можно изобразить таким образом:

Другими словами, произведение элементов в первом определителе, которые соединены прямыми, берется со знаком «+»; так же, для 2го определителя — соответствующие произведения берутся со знаком «-«, то есть по такой схеме:

При решении матриц правилом Саррюса , справа от определителя дописывают первые 2 столбца и произведения соответствующих элементов на главной диагонали и на диагоналях, которые ей параллельны, берут со знаком «+»; а произведения соответствующих элементов побочной диагонали и диагоналей, которые ей параллельны, со знаком «-«:

Разложение определителя по строке или столбцу при решении матриц.

Определитель равен сумме произведений элементов строки определителя на их алгебраические дополнения. Обычно выбирают ту строку/столбец, в которой/ом есть нули. Строку либо столбец, по которой/ому ведется разложение, будут обозначать стрелкой.

Приведение определителя к треугольному виду при решении матриц.

При решении матриц методом приведения определителя к треугольному виду, работают так: с помощью простейших преобразований над строками либо столбцами, определитель становится треугольного вида и тогда его значение, в соответствии со свойствами определителя, будет равно произведению элементов, которые стоят на главной диагонали.

Теорема Лапласа при решении матриц.

Решая матрицы по теореме Лапласа, необходимо знать непосредственно саму теорему. Теорема Лапласа: Пусть Δ — это определитель n -го порядка. Выбираем в нем любые k строк (либо столбцов), при условии k n — 1 . В таком случае сумма произведений всех миноров k -го порядка, содержащихся в выбранных k строках (столбцах), на их алгебраические дополнения будет равна определителю.

Решение обратной матрицы.

Последовательность действий для решения обратной матрицы :

  1. Понять, квадратная ли данная матрица. В случае отрицательного ответа становится ясно, что обратной матрицы для нее не может быть.
  2. Вычисляем алгебраические дополнения.
  3. Составляем союзную (взаимную, присоединённую) матрицу C .
  4. Составляем обратную матрицу из алгебраических дополнений: все элементы присоединённой матрицы C делим на определитель начальной матрицы. Итоговая матрица будет искомой обратной матрицей относительно заданной.
  5. Проверяем выполненную работу: умножаем матрицу начальную и полученную матрицы, результатом должна стать единичная матрица.

Решение систем матриц.

Для решения систем матриц наиболее часто используют метод Гаусса.

Метод Гаусса — это стандартный способ решения систем линейных алгебраических уравнений (СЛАУ) и он заключается в том, что последовательно исключаются переменные, т. е., при помощи элементарных изменений систему уравнений доводят до эквивалентной системы треугольного вида и из нее, последовательно, начиная с последних (по номеру), находят каждый элемент системы.

Метод Гаусса является самым универсальным и лучшим инструментом для нахождения решения матриц. Если у системы бесконечное множество решений или система является несовместимой, то ее нельзя решать по правилу Крамера и матричным методом.

Метод Гаусса подразумевает также прямой (приведение расширенной матрицы к ступенчатому виду, т.е. получение нулей под главной диагональю) и обратный (получение нулей над главной диагональю расширенной матрицы) ходы. Прямой ход и есть метод Гаусса, обратный — метод Гаусса-Жордана. Метод Гаусса-Жордана отличается от метода Гаусса лишь последовательностью исключения переменных.

формула, решения, свойства. Сложение и вычитание матриц

Данное методическое пособие поможет Вам научиться выполнять действия с матрицами : сложение (вычитание) матриц, транспонирование матрицы, умножение матриц, нахождение обратной матрицы. Весь материал изложен в простой и доступной форме, приведены соответствующие примеры, таким образом, даже неподготовленный человек сможет научиться выполнять действия с матрицами. Для самоконтроля и самопроверки Вы можете бесплатно скачать матричный калькулятор >>> .

Я буду стараться минимизировать теоретические выкладки, кое-где возможны объяснения «на пальцах» и использование ненаучных терминов. Любители основательной теории, пожалуйста, не занимайтесь критикой, наша задача – научиться выполнять действия с матрицами .

Для СВЕРХБЫСТРОЙ подготовки по теме (у кого «горит») есть интенсивный pdf-курс Матрица, определитель и зачёт!

Матрица – это прямоугольная таблица каких-либо элементов . В качестве элементов мы будем рассматривать числа, то есть числовые матрицы. ЭЛЕМЕНТ – это термин. Термин желательно запомнить, он будет часто встречаться, не случайно я использовал для его выделения жирный шрифт.

Обозначение: матрицы обычно обозначают прописными латинскими буквами

Пример: рассмотрим матрицу «два на три»:

Данная матрица состоит из шести элементов :

Все числа (элементы) внутри матрицы существуют сами по себе, то есть ни о каком вычитании речи не идет:

Это просто таблица (набор) чисел!

Также договоримся не переставлять числа, если иного не сказано в объяснениях. У каждого числа свое местоположение, и перетасовывать их нельзя!

Рассматриваемая матрица имеет две строки:

и три столбца:

СТАНДАРТ : когда говорят о размерах матрицы, то сначала указывают количество строк, а только потом – количество столбцов. Мы только что разобрали по косточкам матрицу «два на три».

Если количество строк и столбцов матрицы совпадает, то матрицу называют квадратной , например: – матрица «три на три».

Если в матрице один столбец или одна строка , то такие матрицы также называют векторами .

На самом деле понятие матрицы мы знаем еще со школы, рассмотрим, например точку с координатами «икс» и «игрек»: . По существу, координаты точки записаны в матрицу «один на два». Кстати, вот Вам и пример, почему порядок чисел имеет значение: и – это две совершенно разные точки плоскости.

Теперь переходим непосредственно к изучению действий с матрицами :

1) Действие первое. Вынесение минуса из матрицы (внесение минуса в матрицу) .

Вернемся к нашей матрице . Как вы наверняка заметили, в данной матрице слишком много отрицательных чисел. Это очень неудобно с точки зрения выполнения различных действий с матрицей, неудобно писать столько минусов, да и просто в оформлении некрасиво выглядит.

Вынесем минус за пределы матрицы, сменив у КАЖДОГО элемента матрицы знак :

У нуля, как Вы понимаете, знак не меняется, ноль – он и в Африке ноль.

Обратный пример: . Выглядит безобразно.

Внесем минус в матрицу, сменив у КАЖДОГО элемента матрицы знак :

Ну вот, гораздо симпатичнее получилось. И, самое главное, выполнять какие-либо действия с матрицей будет ПРОЩЕ. Потому что есть такая математическая народная примета: чем больше минусов – тем больше путаницы и ошибок .

2) Действие второе. Умножение матрицы на число .

Пример:

Всё просто, для того чтобы умножить матрицу на число, нужно каждый элемент матрицы умножить на данное число. В данном случае – на тройку.

Еще один полезный пример:

– умножение матрицы на дробь

Сначала рассмотрим то, чего делать НЕ НАДО :

Вносить дробь в матрицу НЕ НУЖНО, во-первых, это только затрудняет дальнейшие действия с матрицей, во-вторых, затрудняет проверку решения преподавателем (особенно, если – окончательный ответ задания).

И, тем более, НЕ НАДО делить каждый элемент матрицы на минус семь:

Из статьи Математика для чайников или с чего начать , мы помним, что десятичных дробей с запятой в высшей математике стараются всячески избегать.

Единственное, что желательно сделать в этом примере – это внести минус в матрицу:

А вот если бы ВСЕ элементы матрицы делились на 7 без остатка , то тогда можно (и нужно!) было бы поделить.

Пример:

В этом случае можно и НУЖНО умножить все элементы матрицы на , так как все числа матрицы делятся на 2 без остатка .

Примечание: в теории высшей математики школьного понятия «деление» нет. Вместо фразы «это поделить на это» всегда можно сказать «это умножить на дробь». То есть, деление – это частный случай умножения.

3) Действие третье. Транспонирование матрицы .

Для того чтобы транспонировать матрицу, нужно ее строки записать в столбцы транспонированной матрицы.

Пример:

Транспонировать матрицу

Строка здесь всего одна и, согласно правилу, её нужно записать в столбец:

– транспонированная матрица.

Транспонированная матрица обычно обозначается надстрочным индексом или штрихом справа вверху.

Пошаговый пример:

Транспонировать матрицу

Сначала переписываем первую строку в первый столбец:

Потом переписываем вторую строку во второй столбец:

И, наконец, переписываем третью строку в третий столбец:

Готово. Грубо говоря, транспонировать – это значит повернуть матрицу набок.

4) Действие четвертое. Сумма (разность) матриц .

Сумма матриц действие несложное.
НЕ ВСЕ МАТРИЦЫ МОЖНО СКЛАДЫВАТЬ. Для выполнения сложения (вычитания) матриц, необходимо, чтобы они были ОДИНАКОВЫМИ ПО РАЗМЕРУ.

Например, если дана матрица «два на два», то ее можно складывать только с матрицей «два на два» и никакой другой!

Пример:

Сложить матрицы и

Для того чтобы сложить матрицы, необходимо сложить их соответствующие элементы :

Для разности матриц правило аналогичное, необходимо найти разность соответствующих элементов .

Пример:

Найти разность матриц ,

А как решить данный пример проще, чтобы не запутаться? Целесообразно избавиться от лишних минусов, для этого внесем минус в матрицу :

Примечание: в теории высшей математики школьного понятия «вычитание» нет. Вместо фразы «из этого вычесть это» всегда можно сказать «к этому прибавить отрицательное число». То есть, вычитание – это частный случай сложения.

5) Действие пятое. Умножение матриц .

Какие матрицы можно умножать?

Чтобы матрицу можно было умножить на матрицу нужно, чтобы число столбцов матрицы равнялось числу строк матрицы .

Пример:
Можно ли умножить матрицу на матрицу ?

Значит, умножать данные матрицы можно.

А вот если матрицы переставить местами, то, в данном случае, умножение уже невозможно!

Следовательно, выполнить умножение невозможно:

Не так уж редко встречаются задания с подвохом, когда студенту предлагается умножить матрицы, умножение которых заведомо невозможно.

Следует отметить, что в ряде случаев можно умножать матрицы и так, и так.
Например, для матриц, и возможно как умножение , так и умножение

Сложение матриц:

Вычитание и сложение матриц сводится к соответствующим операциям над их элементами. Операция сложения матриц вводится только для матриц одинакового размера, т. е. для матриц , у которых число строк и столбцов соответственно равно. Суммой матриц А и В, называется матрица С, элементы которой равны сумме соответствующих элементов. С = А + В c ij = a ij + b ij Аналогично определяется разность матриц .

Умножение матрицы на число:

Операция умножения (деления) матрицы любого размера на произвольное число сводится к умножению (делению) каждого элемента матрицы на это число. Произведением матрицы А на число k называется матрица В, такая что

b ij = k × a ij . В = k × A b ij = k × a ij . Матрица — А = (-1) × А называется противоположной матрице А.

Свойства сложения матриц и умножения матрицы на число:

Операции сложения матриц и умножения матрицы на число обладают следующими свойствами: 1. А + В = В + А; 2. А + (В + С) = (А + В) + С; 3. А + 0 = А; 4. А — А = 0; 5. 1 × А = А; 6. α × (А + В) = αА + αВ; 7. (α + β) × А = αА + βА; 8. α × (βА) = (αβ) × А; , где А, В и С — матрицы, α и β — числа.

Умножение матриц (Произведение матриц):

Операция умножения двух матриц вводится только для случая, когда число столбцов первой матрицы равно числу строк второй матрицы . Произведением матрицы А m×n на матрицу В n×p , называется матрица С m×p такая, что с ik = a i1 × b 1k + a i2 × b 2k + … + a in × b nk , т. е. находиться сумма произведений элементов i — ой строки матрицы А на соответствующие элементы j — ого столбца матрицы В. Если матрицы А и В квадратные одного размера, то произведения АВ и ВА всегда существуют. Легко показать, что А × Е = Е × А = А, где А квадратная матрица , Е — единичная матрица того же размера.

Свойства умножения матриц:

Умножение матриц не коммутативно, т.е. АВ ≠ ВА даже если определены оба произведения. Однако, если для каких — либо матриц соотношение АВ=ВА выполняется, то такие матрицы называются перестановочными. Самым характерным примером может служить единичная матрица , которая является перестановочной с любой другой матрицей того же размера. Перестановочными могут быть только квадратные матрицы одного и того же порядка. А × Е = Е × А = А

Умножение матриц обладает следующими свойствами: 1. А × (В × С) = (А × В) × С; 2. А × (В + С) = АВ + АС; 3. (А + В) × С = АС + ВС; 4. α × (АВ) = (αА) × В; 5. А × 0 = 0; 0 × А = 0; 6. (АВ) Т = В Т А Т; 7. (АВС) Т = С Т В Т А Т; 8. (А + В) Т = А Т + В Т;

Определителем матрицы второго порядка, или определителем второго порядка, называется число, которое вычисляется по формуле:

Определителем матрицы третьего порядка, или определителем третьего порядка, называется число, которое вычисляется по формуле:

Это число представляет алгебраическую сумму, состоящую из шести слагаемых. В каждое слагаемое входит ровно по одному элементу из каждой строки и каждого столбца матрицы . Каждое слагаемое состоит из произведения трех сомножителей.

Знаки, с которыми члены определителя матрицы входят в формулу нахождения определителя матрицы третьего порядка можно определить, пользуясь приведенной схемой, которая называется правилом треугольников или правилом Сарруса. Первые три слагаемые берутся со знаком плюс и определяются из левого рисунка, а последующие три слагаемые берутся со знаком минус и определяются из правого рисунка.

Определить количество слагаемых, для нахождения определителя матрицы , в алгебраической сумме, можно вычислив факториал: 2! = 1 × 2 = 2 3! = 1 × 2 × 3 = 6

Свойства определителей матриц

Свойства определителей матриц:

Свойство № 1:

Определитель матрицы не изменится, если его строки заменить столбцами, причем каждую строку столбцом с тем же номером, и наоборот (Транспонирование). |А| = |А| Т

Следствие:

Столбцы и строки определителя матрицы равноправны, следовательно, свойства присущие строкам выполняются и для столбцов.

Свойство № 2:

При перестановке 2-х строк или столбцов определитель матрицы изменит знак на противоположный, сохраняя абсолютную величину, т.е.:

Свойство № 3:

Определитель матрицы , имеющий два одинаковых ряда, равен нулю.

Свойство № 4:

Общий множитель элементов какого-либо ряда определителя матрицы можно вынести за знак определителя .

Следствия из свойств № 3 и № 4:

Если все элементы некоторого ряда (строки или столбца) пропорциональны соответствующим элементам параллельного ряда, то такой определитель матрицы равен нулю.

Свойство № 5:

определителя матрицы равны нулю, то сам определитель матрицы равен нулю.

Свойство № 6:

Если все элементы какой–либо строки или столбца определителя представлены в виде суммы 2-х слагаемых, то определитель матрицы можно представить в виде суммы 2-х определителей по формуле:

Свойство № 7:

Если к какой–либо строке (или столбцу) определителя прибавить соответствующие элементы другой строки (или столбца), умноженные на одно и тоже число, то определитель матрицы не изменит своей величины.

Пример применения свойств для вычисления определителя матрицы :

Это одна из самых распространенных операций с матрицами. Матрица, которая получается после умножения, называется произведением матриц.

Произведением матрицы A m × n на матрицу B n × k будет матрица C m × k такая, что элемент матрицы C , находящийся в i -ой строке и j -ом столбце, то есть элемент c ij равен сумме произведений элементов i -ой строки матрицы A на соответствующие элементы j -ого столбца матрицы B .

Процесс умножения матриц возможен только в случае, когда число столбцов первой матрицы равно числу строк второй матрицы.

Пример:
Можно ли умножить матрицу на матрицу ?

m = n , значит, умножать данные матрицы можно.

Если же матрицы поменять местами, то, при таких матрицах, умножение уже не будет возможно.

m n , таким образом, выполнять умножение нельзя:

Довольно часто можно встретить задания с подвохом, когда ученику предлагается умножить матрицы , умножение которых заведомо невозможно.

Обратите внимание, что иногда можно умножать матрицы и так, и так. К примеру, для матриц, и возможно как умножение MN , так и умножение NM.

Это не очень сложное действие. Умножение матриц лучше понимать на конкретных примерах, т.к. только определение может сильно запутать.

Начнем с самого простого примера:

Необходимо умножить на . Первым делом приведем формулу для данного случая:

— здесь хорошо прослеживается закономерность.

Умножить на .

Формула для этого случая: .

Умножение матриц и результат:

В результате получена т.н. нулевая матрица.

Очень важно помнить, что здесь не работает «правило перестановки мест слагаемых» так как почти всегда MN NM . Поэтому, производя операцию умножения матриц их ни в коем случае нельзя менять местами.

Теперь рассмотрим примеры умножения матриц третьего порядка:

Умножить на .

Формула очень похожа на прошлые:

Решение матрицы: .

Это тоже самое умножение матриц, только вместо второй матрицы берется простое число. Как можно догадаться, такое умножение выполнять гораздо проще.

Пример умножения матрицы на число:

Тут все понятно — для того, чтобы умножить матрицу на число , необходимо каждый элемент матрицы последовательно умножить на указанное число. В данном случае — на 3.

Еще один полезный пример:

— умножение матрицы на дробное число.

Первым делом покажем то, чего делать не надо:

При умножении матрицы на дробное число не нужно вносить дробь в матрицу, так как это в первую очередь только затрудняет дальнейшие действия с матрицей, во-вторых, затрудняет проверку решения преподавателем.

И, тем более, не нужно делить каждый элемент матрицы на -7:

.

Что стоит сделать в данном случае — это внести минус в матрицу:

.

Если бы у вас был пример, когда все элементы матрицы делились бы на 7 без остатка, то тогда можно (и нужно!) было бы поделить.

В данном примере можно и нужно умножить все элементы матрицы на ½, т.к. каждый элемент матрицы делится на 2 без остатка.

Примечание: в теории высшей математики школьного понятия «деление» нет. Вместо фразы «это поделить на это» всегда можно сказать «это умножить на дробь». То есть, деление — это частный случай умножения.

Будем последовательно “исключать” неизвестные. Для этого первое уравнение системы оставим без изменений, а второе и третье преобразуем:

1) ко второму уравнению прибавим первое, умноженное на –2, и приведем его к виду –3x 2 –2x 3 = –2;

2) к третьему уравнению прибавим первое, умноженное на – 4, и приведем его к виду –3x 2 – 4x 3 = 2.

В результате из второго и третьего уравнений будет исключено неизвестное x 1 и система примет вид

Второе и третье уравнения системы умножим на –1, получим

Коэффициент 1 в первом уравнении при первом неизвестном х 1 называется ведущим элементом первого шага исключения.

На втором шаге первое и второе уравнения остаются без изменений, а к третьему уравнению применим тот же способ исключения переменной x 2 . Ведущим элементом второго шага является коэффициент 3. К третьему уравнению прибавим второе, умноженное на –1, тогда система преобразуется к виду

(1.2)

Процесс приведения системы (1.1) к виду (1.2) называются прямым ходом метода Гаусса.

Порядок действий решения системы (1.2) называется обратным ходом. Из последнего уравнения получим х 3 = –2. Подставляя это значение во второе уравнение, получим х 2 = 2. После этого первое уравнение дает х 1 = 1. Таким образом, — решение системы (1.1).

Понятие матрицы

Рассмотрим величины, входящие в систему (1.1). Набор из девяти числовых коэффициентов, стоящих в уравнениях перед неизвестными, образует таблицу чисел, которая называется матрицей :

А = . (1.3)

Числа таблицы называются элементами матрицы. Элементы образуют строки и столбцы матрицы. Количество строк и количество столбцов образуют размерность матрицы. Матрица А имеет размерность 3´3 (“три на три”), причем первое число указывает количество строк, а второе – столбцов. Часто матрицу обозначают, указывая ее размерность А (3 ´ 3) . Так как число строк и столбцов в матрице А одинаково, матрица называется квадратной. Количество строк (и столбцов) в квадратной матрице называется ее порядком , поэтому А – матрица третьего порядка .

Правые части уравнений, также образуют таблицу чисел, т.е. матрицу:

Каждая строка этой матрицы образована единственным элементом, поэтому B (3 ´ 1) называется матрицей–столбцом , ее размерность 3´1. Набор неизвестных также можно представить как матрицу-столбец:

Умножение квадратной матрицы на матрицу-столбец

С матрицами можно производить различные операции, которые будут подробно рассмотрены в дальнейшем. Здесь же разберем только правило умножения квадратной матрицы на матрицу-столбец. По определению , результатом умножения матрицы А (3 ´ 3) на столбец В (3 ´ 1) является столбец D (3 ´ 1) , элементы которого равны суммам произведений элементов строк матрицы А на элементы столбца В :

2)второй элемент столбца D равен сумме произведений элементов второй строки матрицы А на элементы столбца В :

Из приведенных формул видно, что умножить матрицу на столбец В можно только в случае, если число столбцов матрицы А равно числу элементов в столбце В .

Рассмотрим еще два числовых примера умножения матрицы (3 ´3) на столбец (3 ´1) :

Пример 1.1

АВ = .

Пример 1.2

АВ = .

Определение. Произведением двух матриц А и В называется матрица С , элемент которой, находящийся на пересечении i -й строки и j -го столбца, равен сумме произведений элементов i -й строки матрицы А на соответствующие (по порядку) элементы j -го столбца матрицы В .

Из этого определения следует формула элемента матрицы C :

Произведение матрицы А на матрицу В обозначается АВ .

Пример 1. Найти произведение двух матриц А и B , если

,

.

Решение. Удобно нахождение произведения двух матриц А и В записывать так, как на рис.2:

На схеме серые стрелки показывают, элементы какой строки матрицы А на элементы какого столбца матрицы В нужно перемножить для получения элементов матрицы С , а линиями цвета элемента матрицы C соединены соответствующие элементы матриц A и B , произведения которых складываются для получения элемента матрицы C .

В результате получаем элементы произведения матриц:

Теперь у нас есть всё, чтобы записать произведение двух матриц:

.

Произведение двух матриц АВ имеет смысл только в том случае, когда число столбцов матрицы А совпадает с числом строк матрицы В .

Эту важную особенность будет легче запомнить, если почаще пользоваться следующими памятками:

Имеет место ещё одна важная особенность произведения матриц относительно числа строк и столбцов:

В произведении матриц АВ число строк равно числу строк матрицы А , а число столбцов равно числу столбцов матрицы В .

Пример 2. Найти число строк и столбцов матрицы C , которая является произведением двух матриц A и B следующих размерностей:

а) 2 Х 10 и 10 Х 5;

б) 10 Х 2 и 2 Х 5;

Пример 3. Найти произведение матриц A и B , если:

.

A B — 2. Следовательно, размерность матрицы C = AB — 2 X 2.

Вычисляем элементы матрицы C = AB .

Найденное произведение матриц: .

Проверить решение этой и других подобных задач можно на калькуляторе произведения матриц онлайн .

Пример 5. Найти произведение матриц A и B , если:

.

Решение. Число строк в матрице A — 2, число столбцов в матрице B C = AB — 2 X 1.

Вычисляем элементы матрицы C = AB .

Произведение матриц запишется в виде матрицы-столбца: .

Проверить решение этой и других подобных задач можно на калькуляторе произведения матриц онлайн .

Пример 6. Найти произведение матриц A и B , если:

.

Решение. Число строк в матрице A — 3, число столбцов в матрице B — 3. Следовательно, размерность матрицы C = AB — 3 X 3.

Вычисляем элементы матрицы C = AB .

Найденное произведение матриц: .

Проверить решение этой и других подобных задач можно на калькуляторе произведения матриц онлайн .

Пример 7. Найти произведение матриц A и B , если:

.

Решение. Число строк в матрице A — 1, число столбцов в матрице B — 1. Следовательно, размерность матрицы C = AB — 1 X 1.

Вычисляем элемент матрицы C = AB .

Произведение матриц является матрицей из одного элемента: .

Проверить решение этой и других подобных задач можно на калькуляторе произведения матриц онлайн .

Программная реализация произведения двух матриц на С++ разобрана в соответствующей статье в блоке «Компьютеры и программирование».

Возведение матрицы в степень

Возведение матрицы в степень определяется как умножение матрицы на ту же самую матрицу. Так как произведение матриц существует только тогда, когда число столбцов первой матрицы совпадает с числом строк второй матрицы, то возводить в степень можно только квадратные матрицы. n -ая степень матрицы путём умножения матрицы на саму себя n раз:

Пример 8. Дана матрица . Найти A ² и A ³ .

Найти произведение матриц самостоятельно, а затем посмотреть решение

Пример 9. Дана матрица

Найти произведение данной матрицы и транспонированной матрицы , произведение транспонированной матрицы и данной матрицы.

Свойства произведения двух матриц

Свойство 1. Произведение любой матрицы А на единичную матрицу Е соответствующего порядка как справа, так и слева, совпадает с матрицей А, т.е. АЕ = ЕА = А.

Иными словами, роль единичной матрицы при умножении матриц такая же, как и единицы при умножении чисел.

Пример 10. Убедиться в справедливости свойства 1, найдя произведения матрицы

на единичную матрицу справа и слева.

Решение. Так как матрица А содержит три столбца, то требуется найти произведение АЕ , где


единичная матрица третьего порядка. Найдём элементы произведения С = АЕ :

Получается, что АЕ = А .

Теперь найдём произведение ЕА , где Е – единичная матрица второго порядка, так как матрица А содержит две строки. Найдём элементы произведения С = ЕА :

3.2: Свойства определителей — Mathematics LibreTexts

  1. Последнее обновление
  2. Сохранить как PDF
  • Идентификатор страницы
    14511
    • Кен Каттлер
    • Университет Бригама Янга via Lyryx

    Свойства определителей I: примеры

    У определителей много важных свойств. Поскольку многие из этих свойств связаны с операциями со строками, которые обсуждались в главе 1, мы сейчас вспомним это определение.

    Определение \(\PageIndex{1}\): Операции со строками

    Операции со строками состоят из следующих

    1. Переключение двух строк.
    2. Умножить строку на ненулевое число.
    3. Заменить строку числом, кратным другой строке, добавленной к самой себе.

    Теперь рассмотрим влияние операций над строками на определитель матрицы. В следующих разделах мы увидим, что использование следующих свойств может сильно помочь в поиске определителей. В этом разделе теоремы будут использоваться в качестве мотивации для предоставления различных примеров полезности свойств.

    Первая теорема объясняет влияние на определитель матрицы перестановки двух строк.

    Теорема \(\PageIndex{1}\): переключение строк

    Пусть \(A\) — матрица \(n\times n\), а \(B\) — матрица, полученная в результате перестановки двух строк \(A. \). Тогда \(\det \left( B \right) = — \det \left( A\right).\)

    Когда мы переставляем две строки матрицы, определитель умножается на \(-1\). Рассмотрим следующий пример.

    Пример \(\PageIndex{1}\): переключение двух строк

    Пусть \(A=\left[ \begin{array}{rr} 1 & 2 \\ 3 & 4 \end{array} \right] \) и пусть \(B=\left[ \begin{array}{rr} 3 & 4 \\ 1 & 2 \end{array} \right]\). Зная, что \(\det \left( A \right) = -2\), найдите \(\det \left( B \right)\).

    Решение

    По определению 3.1.1, \(\det \left(A\right) = 1 \times 4 — 3 \times 2 = -2\). Обратите внимание, что строки \(B\) являются строками \(A\), но перепутаны. По теореме \(\PageIndex{1}\), так как две строки \(A\) были переставлены местами, \(\det \left(B\right) = — \det \left(A\right) = — \left (-2\справа) = 2\). Вы можете убедиться в этом, используя Определение 3.1.1.

    Следующая теорема демонстрирует влияние на определитель матрицы, когда мы умножаем строку на скаляр. 9п \дет(А)\).

    Рассмотрим следующий пример.

    Пример \(\PageIndex{2}\): умножение строки на 5

    Пусть \(A=\left[ \begin{array}{rr} 1 & 2 \\ 3 & 4 \end{array} \ right] ,\ B=\left[ \begin{array}{rr} 5 & 10 \\ 3 & 4 \end{array} \right].\) Зная, что \(\det \left( A \right) = -2\), найти \(\det \left( B \right)\).

    Решение

    По определению 3.1.1, \(\det \left( A\right) = -2.\) Мы также можем вычислить \(\det \left(B\right)\), используя определение 3.1 .1, и мы видим, что \(\det \left(B\right) = -10\).

    Теперь давайте вычислим \(\det \left(B\right)\), используя теорему \(\PageIndex{2}\), и посмотрим, получим ли мы тот же ответ. Обратите внимание, что первая строка \(B\) в \(5\) раз больше первой строки \(A\), а вторая строка \(B\) равна второй строке \(A\) . По теореме \(\PageIndex{2}\), \(\det \left( B \right) = 5 \times \det \left( A \right) = 5 \times -2 = -10.\)

    Вы можете видеть, что это соответствует нашему ответу выше.

    Наконец, рассмотрим следующую теорему для последней операции со строками — прибавления числа, кратного одной строке, к другой строке.

    Теорема \(\PageIndex{4}\): добавление кратного строки к другой строке матрица, которая получается в результате прибавления числа, кратного одной строке, к другой строке. Тогда \(\det \left( A\right) =\det \left( B \right)\).

    Следовательно, когда мы прибавляем кратное одной строки к другой строке, определитель матрицы не меняется. Обратите внимание, что если матрица \(A\) содержит строку, кратную другой строке, \(\det \left(A\right)\) будет равно \(0\). Чтобы убедиться в этом, предположим, что первая строка \(A\) равна \(-1\), умноженной на вторую строку. По теореме \(\PageIndex{4}\) мы можем добавить первую строку ко второй строке, и определитель не изменится. Однако эта операция строки приведет к строке нулей. Используя разложение Лапласа по ряду нулей, мы находим, что определитель равен \(0\).

    Рассмотрим следующий пример.

    Пример \(\PageIndex{3}\): добавление строки к другой строке

    Пусть \(A=\left[ \begin{array}{rr} 1 & 2 \\ 3 & 4 \end{array} \right]\) и пусть \(B=\left[ \begin{array}{rr} 1 & 2 \\ 5 & 8 \end{array} \right] . \) Найти \(\det \left(B \Правильно)\).

    Решение

    По определению 3.1.1 \(\det \left(A\right) = -2\). Обратите внимание, что вторая строка \(B\) в два раза больше первой строки \(A\), добавленной ко второй строке. По теореме \(\PageIndex{1}\), \(\det \left( B\right) = \det \left( A \right) = -2\). Как обычно, вы можете проверить этот ответ, используя Определение 3.1.1.

    Пример \(\PageIndex{4}\): Несколько строк

    Пусть \(A = \left[ \begin{array}{rr} 1 & 2 \\ 2 & 4 \end{array} \right ]\). Покажите, что \(\det \left( A \right) = 0\).

    Решение

    Используя определение 3.1.1, определитель имеет вид \[\det \left( A \right) = 1 \times 4 — 2 \times 2 = 0\nonumber \]

    Однако обратите внимание, что вторая строка равна \(2\), умноженной на первую строку. Тогда по приведенному выше обсуждению в соответствии с теоремой \(\PageIndex{4}\) определитель будет равен \(0\).

    До сих пор основное внимание уделялось операциям со строками. Однако мы можем выполнять те же операции со столбцами, а не со строками. Три операции, описанные в определении \(\PageIndex{1}\), можно выполнять со столбцами, а не со строками. В этом случае в теоремах \(\PageIndex{1}\), \(\PageIndex{2}\) и \(\PageIndex{4}\) слово «строка» можно заменить словом «столбец». «.

    Есть несколько других важных свойств определителей, которые не включают операции со строками (или столбцами). Первым является определитель произведения матриц.

    Теорема \(\PageIndex{5}\): определитель произведения

    Пусть \(A\) и \(B\) — две матрицы \(n\times n\). Тогда \[\det \left( AB\right) =\det \left( A\right) \det \left( B\right)\nonumber \]

    Чтобы найти определитель произведения матриц, мы можно просто взять произведение определителей.

    Рассмотрим следующий пример.

    Пример \(\PageIndex{5}\): определитель произведения

    Сравните \(\det \left( AB\right)\) и \(\det \left( A\right) \det \left ( B\right)\) для \[A=\left[ \begin{array}{rr} 1 & 2 \\ -3 & 2 \end{array} \right], B=\left[ \begin{array }{rr} 3 и 2 \\ 4 и 1 \end{массив} \right]\nonumber \]

    Решение

    Первое вычисление \(AB\), которое задается как \[AB=\left[ \begin{array}{rr} 1 & 2 \\ -3 & 2 \end{array} \right ] \left[ \begin{array}{rr} 3 & 2 \\ 4 & 1 \end{array} \right] = \left[ \begin{array}{rr} 11 & 4 \\ -1 & -4 \end{array} \right]\nonumber \] и, следовательно, по определению 3. 1.1 \[\det \left( AB\right) =\det \left[ \begin{array}{rr} 11 & 4 \\ — 1 & -4 \end{array} \right] = -40\nonumber \]

    Теперь \[\det \left( A\right) =\det \left[ \begin{array}{rr} 1 & 2 \\ -3 & 2 \end{массив} \right] = 8\nonumber \] и \[\det \left( B\right) =\det \left[ \begin{array}{rr} 3 & 2 \ \ 4 & 1 \ конец {массив} \ справа] = -5 \ не число \] 9{-1}) = \frac{1}{\det(A)}\nonumber \]

    Рассмотрим следующий пример.

    Пример \(\PageIndex{7}\): Определитель обратимой матрицы

    Пусть \(A = \left[ \begin{array}{rr} 3 & 6 \\ 2 & 4 \end{array} \ справа], B = \left[ \begin{array}{rr} 2 & 3 \\ 5 & 1 \end{array} \right]\). Для каждой матрицы определите, является ли она обратимой. Если да, то найти определитель обратного.

    Решение

    Сначала рассмотрим матрицу \(A\). Используя определение 3.1.1, мы можем найти определитель следующим образом: \[\det \left( A \right) = 3 \times 4 — 2 \times 6 = 12 — 12 = 0\nonnumber \] {7}\) \(А\) необратима. 9n a_{1,i} \mathrm{cof}(A)_{1,i}.\] Если \(n=1\), то \(\det A=a_{1,1}\).

    Следующий пример является простым и настоятельно рекомендуется в качестве средства для привыкания к определениям.

    Пример \(\PageIndex{8}\):

    (1) Пусть \(E_{ij}\) — элементарная матрица, полученная перестановкой \(i\)-й и \(j\)-й строк матрицы \ (Я\). Тогда \(\det E_{ij}=-1\).

    (2) Пусть \(E_{ik}\) — элементарная матрица, полученная умножением \(i\)-й строки \(I\) на \(k\). Тогда \(\det E_{ik}=k\). 9Т\).

    Многие доказательства в этом разделе используют принцип математической индукции. Эта концепция обсуждается в Приложении A.2 и для удобства рассмотрена здесь. Сначала проверим, что утверждение верно для \(n=2\) (случай \(n=1\) либо совсем тривиален, либо бессмыслен).

    Далее предположим, что утверждение верно для \(n-1\) (где \(n\geq 3\)) и докажем его для \(n\). Как только это будет выполнено, по принципу математической индукции мы можем заключить, что утверждение верно для всех матриц \(n\times n\) для каждого \(n\geq 2\).

    Если \(A\) является \(n\times n\) матрицей и \(1\leq j \leq n\), то матрица, полученная удалением \(1\)st столбца и \(j\ )-я строка из \(A\) является матрицей \(n-1\times n-1\) (ниже мы будем обозначать эту матрицу через \(A(j)\)). Поскольку эти матрицы используются при вычислении кофакторов \(\mathrm{cof}(A)_{1,i}\), для \(1\leq i\neq n\), к этим матрицам применимо предположение индукции.

    Рассмотрим следующую лемму.

    Лемма \(\PageIndex{1}\):

    Если \(A\) — матрица \(n\times n\) такая, что одна из ее строк состоит из нулей, то \(\det A=0 \).

    Доказательство

    Мы докажем эту лемму с помощью математической индукции.

    Если \(n=2\) это просто (проверьте!).

    Пусть \(n\geq 3\) таково, что каждая матрица размера \(n-1\times n-1\) со строкой, состоящей из нулей, имеет определитель, равный нулю. Пусть \(i\) таково, что \(i\)-я строка \(A\) состоит из нулей. Тогда мы имеем \(a_{ij}=0\) для \(1\leq j\leq n\).

    Исправьте \(j\in \{1,2, \dots ,n\}\) так, чтобы \(j\neq i\). Тогда матрица \(A(j)\), используемая при вычислении \(\mathrm{cof}(A)_{1,j}\), имеет строку, состоящую из нулей, и по нашему индуктивному предположению \(\mathrm{cof }(A)_{1,j}=0\). 9n a_{1,j} \mathrm{cof}(A)_{1,j}=0\nonumber \], так как каждое из слагаемых равно 0,

    Лемма \(\PageIndex{2}\):

    Предположим, что \(A\), \(B\) и \(C\) являются \(n\×n\) матрицами, которые для некоторого \(1\ leq i\leq n\) удовлетворяют следующему.

    1. \(j\)-е строки всех трех матриц одинаковы, для \(j\neq i\).
    2. Каждая запись в \(j\)-й строке \(A\) является суммой соответствующих записей в \(j\)-х строках \(B\) и \(C\).

    Тогда \(\det A=\det B+\det C\).

    Доказательство

    Проверить на \(n=2\) несложно (проверьте!).

    Теперь предположим, что утверждение леммы верно для \(n-1\times n-1\) матриц, и зафиксируем \(A,B\) и \(C\), как в утверждении. Предположения утверждают, что мы имеем \(a_{l,j}=b_{l,j}=c_{l,j}\) для \(j\neq i\) и для \(1\leq l\leq n \) и \(a_{l,i}=b_{l,i}+c_{l,i}\) для всех \(1\leq l\leq n\). Поэтому \(A(i)=B(i)=C(i)\), а \(A(j)\) обладает тем свойством, что его \(i\)-я строка является суммой \(i\) строки \(B(j)\) и \(C(j)\) для \(j\neq i\), а остальные строки всех трех матриц идентичны. Следовательно, согласно нашему индуктивному предположению, мы имеем \(\mathrm{cof}(A)_{1j}=\mathrm{cof}(B)_{1j}+\mathrm{cof}(C)_{1j}\) для \(j\neq i\). 9n a_{1,l} \mathrm{cof}(A)_{1,l}\\ &=\sum_{l\neq i} a_{1,l}(\mathrm{cof}(B)_{ 1,l}+\mathrm{cof}(C)_{1,l})+ (b_{1,i}+c_{1,i})\mathrm{cof}(A)_{1,i} \\ &= \det B+\det C\end{aligned}\] Это доказывает, что утверждение верно для всех \(n\), и завершает доказательство.

    Теорема \(\PageIndex{8}\):

    Пусть \(A\) и \(B\) — матрицы \(n\times n\).

    1. Если \(A\) получается перестановкой \(i\)-й и \(j\)-й строк \(B\) (с \(i\neq j\)), то \(\det А=-\det В\).
    2. Если \(A\) получается умножением \(i\)-й строки \(B\) на \(k\), то \(\det A=k\det B\).
    3. Если две строки \(A\) идентичны, то \(\det A=0\).
    4. Если \(A\) получается умножением \(i\)-й строки \(B\) на \(k\) и прибавлением его к \(j\)-й строке \(B\) (\( i\neq j\)) затем \(\det A=\det B\).
    Доказательство

    Докажем все утверждения по индукции. Случай \(n=2\) легко проверить напрямую (и настоятельно рекомендуется проверить его).

    Мы предполагаем, что \(n\geq 3\) и (1)–(4) верны для всех матриц размера \(n-1\times n-1\).

    (1) Докажем случай, когда \(j=i+1\), т. е. мы меняем местами две последовательные строки.

    Пусть \(l\in \{1, \dots, n\}\setminus \{i,j\}\). Тогда \(A(l)\) получается из \(B(l)\) перестановкой двух его строк (рисунок) и по нашему предположению \[\label{E2} \mathrm{cof}(A) _{1,l}=-\mathrm{cof}(B)_{1,l}.\]

    Теперь рассмотрим \(a_{1,i} \mathrm{cof}(A)_{1,l}\). n b_{1l} B_{1l} =\det B.\nonumber \]

    Таким образом, мы доказали случай (1), когда \(j=i+1\). Для доказательства общего случая понадобится следующий факт. Если \(i

    (2) Это похоже на (1)… но гораздо проще. Предположим, что (2) верно для всех \(n-1\times n-1\) матриц. У нас есть это \(a_{ji}=k b_{ji}\) для \(1\leq j\leq n\). В частности \(a_{1i}=kb_{1i}\), а при \(l\neq i\) матрица \(A(l)\) получается из \(B(l)\) умножением одного из его строки на \(k\). Поэтому \(\mathrm{cof}(A)_{1l}=k\mathrm{cof}(B)_{1l}\) для \(l\neq i\), и для всех \(l\) мы имеют \(a_{1l} \mathrm{cof}(A)_{1l}=k b_{1l}\mathrm{cof}(B)_{1l}\). По \(\eqref{E1}\) имеем \(\det A=k\det B\).

    (3) Это следствие (1). Если две строки \(A\) идентичны, то \(A\) равна матрице, полученной путем перестановки этих двух строк и, следовательно, согласно (1) \(\det A=-\det A\). Отсюда следует \(\det A=0\).

    (4) Предположим, что (4) верно для всех \(n-1\times n-1\) матриц, и зафиксируем \(A\) и \(B\) так, что \(A\) получается умножением \ (i\)-я строка \(B\) на \(k\) и добавление ее к \(j\)-й строке \(B\) (\(i\neq j\)) затем \(\det А=\дет В\). Если \(k=0\), то \(A=B\) и доказывать нечего, поэтому можно считать \(k\neq 0\).

    Пусть \(C\) — матрица, полученная заменой \(j\)-й строки \(B\) на \(i\)-ю строку \(B\), умноженной на \(k\). По лемме \(\PageIndex{2}\) мы имеем, что \[\det A=\det B+\det C\nonumber \], и нам «всего лишь» нужно показать, что \(\det C=0\). Но \(i\)-я и \(j\)-я строки \(C\) пропорциональны. Если \(D\) получается умножением \(j\)-й строки \(C\) на \(\frac 1k\), то по (2) имеем \(\det C=\frac 1k\det D\) (напомним, что \(k\neq 0\)!). Но \(i\)-я и \(j\)-я строки \(D\) идентичны, поэтому по (3) мы имеем \(\det D=0\) и, следовательно, \(\det C=0\ ).

    Теорема \(\PageIndex{9}\):

    Пусть \(A\) и \(B\) две матрицы \(n\times n\). Тогда \[\det \left( AB\right) =\det \left( A\right) \det \left( B\right)\nonumber \]

    Proof

    Если \(A\) является элементарной матрицей любого типа, то умножение на \(A\) слева имеет тот же эффект, что и выполнение соответствующей операции с элементарной строкой. Поэтому равенство \(\det (AB) =\det A\det B\) в этом случае следует из примера \(\PageIndex{8}\) и теоремы \(\PageIndex{8}\).

    Если \(C\) является редуцированной ступенчатой ​​формой \(A\), то мы можем написать \(A=E_1\cdot E_2\cdot\dots\cdot E_m\cdot C\) для некоторых элементарных матриц \( E_1,\точки, E_m\).

    Теперь рассмотрим два случая.

    Предположим сначала, что \(C=I\). Затем \(A=E_1\cdot E_2\cdot \dots\cdot E_m\) и \(AB= E_1\cdot E_2\cdot \dots\cdot E_m B\). Применяя приведенное выше равенство \(m\) раз, а затем \(m-1\) раз, мы получаем, что \[\begin{aligned} \det AB&=\det E_1\det E_2\cdot \det E_m\cdot \det B\\ &=\det (E_1\cdot E_2\cdot\dots\cdot E_m) \det B\\ &=\det A\det B. \end{aligned}\]

    Теперь предположим \(C\neq I\). Поскольку он находится в редуцированной строчно-эшелонной форме, его последняя строка состоит из нулей, а по (4) примера \(\PageIndex{8}\) последняя строка \(CB\) состоит из нулей. По лемме \(\PageIndex{1}\) имеем \(\det C=\det (CB)=0\) и, следовательно, \[\det A=\det (E_1\cdot E_2\cdot E_m)\cdot \ det (C) = \det (E_1\cdot E_2\cdot E_m)\cdot 0=0\nonumber \], а также \[\det AB=\det (E_1\cdot E_2\cdot E_m)\cdot \det (C B ) =\det (E_1\cdot E_2\cdot\dots\cdot E_m) 0 =0\nonnumber \] следовательно \(\det AB=0=\det A \det B\). 9Т\).

    Приведенные выше рассуждения позволяют нам теперь доказать теорему 3. n a_{j,i} \mathrm{cof}(A)_{j,i}.\nonumber \] Докажем случай когда \(j=2\). 9T\), что равно разложению сомножителя по столбцу \(1\) матрицы \(A\). Таким образом, доказательство завершено.


    Эта страница под названием 3.2: Свойства детерминантов распространяется в соответствии с лицензией CC BY 4.0 и была создана, изменена и/или курирована Кеном Каттлером (Lyryx) с помощью исходного контента, который был отредактирован в соответствии со стилем и стандартами платформы LibreTexts; подробная история редактирования доступна по запросу.

    1. Наверх
      • Была ли эта статья полезной?
      1. Тип изделия
        Раздел или Страница
        Автор
        Кен Каттлер
        Лицензия
        СС BY
        Версия лицензии
        4,0
        Показать страницу TOC
        нет
      2. Теги
        1. Детерминанты
        2. операции со строками
        3. источник@https://lyryx. com/first-course-linear-алгебра

      Доказательство первой теоремы об определителях

      Доказательство первой теоремы об определителях Вот теорема.

      Теорема. Пусть A будет матрицей n на n . затем выполняются следующие условия.

      1. Если мы умножим строку (столбец) числа A на число, определитель будет умножено на то же число.
      2. Если i -я строка (столбец) в А является суммой i -й строки (столбца) матрицы B и i -й строки (столбца) матрицы C и всех остальных строк в B и C равны соответствующим строкам в A (то есть B и C отличаются от A только одной строкой), тогда det( A )=det( B )+det( C ).
      3. Если две строки (столбца) в A равны, то det( A )=0.
      4. Если мы добавим строку (столбец) числа A , умноженного на скаляр k , к другой строке (столбцу) числа A , то определитель не изменится.
      5. Если поменять местами две строки (столбца) в A , определитель изменится его знак.
      6. det( A )=det( A T ).
      Доказательство. 1. В выражение определителя A каждого продукта содержит ровно одну запись из каждой строки и ровно одну запись из каждой столбец. Таким образом, если мы умножим строку (столбец) на число, скажем, k , каждый член в выражении определителя результирующего матрица будет равна соответствующему члену в det( A ) умножить на k . Поэтому определитель полученной матрицы будет равен k *det( A ).

      2. Имеем, что A(i,j)=B(i,j)+C(i,j) для каждых j =1,2,…,n. Рассмотрим выражение для дет( А ). Каждое слагаемое в этом выражении содержит ровно один множитель из i -й ряд A . Рассмотрим один из этих терминов:

      (-1) р А(р 1 ,1) А(р 2 ,2)…А(i,j). .. А(р n ,n)

      ( A(i,j) — множитель из i -й строки в этом произведении).

      Заменить A(i,j) на B(i,j)+C(i,j) :

      (-1) p A(p 1 ,1) A(p 2 ,2)…(B(i,j)+C(i,j))… A(p н ,н)

      Умножить через:

      (-1) р А(р 1 ,1) А(р 2 ,2)…B(i,j)… A(p n ,n)+
      (-1) p A(p 1 ,1) A(p 2 ,2)…C(i,j)… A(p n ,n)

      Если мы добавим только члены, содержащие B , мы получим определитель B ; если мы сложим все члены, содержащие C , мы получим определитель C . Таким образом det( A )=det( B )+det( C ).

      3. Предположим, что i -я строка в A равна j -я строка в A , то есть A(i,k)=A(j,k) для каждых k =1,2,. .., n . Рассмотрим произвольное произведение в выражение det( A ):

      A(p 1 ,1) A(p 2 ,2)…A(i,k1)…A(j,k2)… A(p n ,n)

      (используем тот факт, что это произведение содержит один множитель из и -й строки и один фактор из j -я строка, и мы предполагаем, что i стоит перед j ; случай, когда i встречается дальше, чем j , аналогичен). Рассмотрим также произведение, соответствующее перестановке p ‘, полученное из p . путем переключения i и j :

      A(p 1 ,1) A(p 2 ,2)…A(j,k1)…A(i,k2)… A(p n ,n)

      Теперь с тех пор

      А(i,k1)=А(j,k1), А(j,k2)=А(i,k2)

      эти условия равны. Но они встречаются в выражении det( A ) с противоположным знаки (помните, что p ‘ получается из p одной транспозицией). Таким образом, эти продукты убивают друг друга в det( A ). Поэтому каждый член в det( A ) уничтожается, когда мы объединяем одинаковые члены в det( A ), поэтому det( A )=0.

      4. Пусть матрица B получена из матрицы A путем сложения j -й строки, умноженной на k до и -го ряда. Представим A в виде столбца строк:

                           [ р  1  ]
                             ...
                           [ р  я  ]
                             ...
                           [ р  дж  ]
                             ...
                           [ р  н  ]
       

      Тогда B имеет следующий вид:

                           [ р  1  ]
                             ...
                           [ г  я  +кр  я  ]
                             ...
                           [ р  дж  ]
                             ...
                           [ р  н  ]
       

      По свойству 2 можно заключить, что det( B ) равно сумме определителей из двух матриц:

                           [ р  1 905:25 ]
                             . ..
                           [ р  я  ]
                             ...
                           [ р  дж  ]
                             ...
                           [ р  н  ]
       

      а также

                           [ р  1  ]
                             ...
                           [ кр  j  ]
                             ...
                           [ р  дж  ]
                             ...
                           [ р  п 905:25 ]
       

      Первая из этих матриц A . Обозначим второй через C . Так det( B )=det( A )+det( C ). По свойству 1 det( C ) в k раз больше определителя следующая матрица:

                           [ р  1  ]
                             ...
                           [ р  дж  ]
                             ...
                           [ р  дж  ]
                             ...
                           [ р  и ]
       

      Но эта матрица имеет две равные строки, поэтому ее определитель равен 0 (свойство 3). Таким образом, det( C )=0 и det( B )=det( A ). Доказательство завершено.

      5. Предположим, что мы поменяли местами i -ю строку и j -ю строку матрицы A . Представьте A в виде столбца строк:

                           [ р  1  ]
                             ...
                           [ р  и ]
                             ...
                           [ р  дж  ]
                             ...
                           [ р  н  ]
       
      Чтобы поменять местами r i и r j , мы можем сделать следующее процедура:
      1. Добавьте j -ю строку к i -й строке:
                             [ р  1  ]
                               ...
                             [ г  я  + г  j  ]
                              ...
                             [ р  и  ]
                               ...
                             [ р  н  ]
         
      2. Вычесть i -ю строку полученной матрицы из j -й строки:
                             [ р  1  ]
                               . ..
                             [ г  я  + г  j  ]
                               ...
                             [-r  я ]
                               ...
                             [ р  и ]
         
      3. Прибавляем j -ю строку полученной матрицы к i -й строке:
                             [ р  1  ]
                               ...
                             [ р  дж  ]
                               ...
                             [-r  я ]
                               ...
                             [ р  н  ]
         
      4. Умножить j -ю строку полученной матрицы на -1:
                             [ р  1  ]
                               ...
                             [ р  дж  ]
                               ...
                             [ р  я  ]
                               ...
                             [ р  н  ]
         

        По свойствам, которые мы уже доказали, все операции этой процедуры кроме самого последнего определитель не меняем. Последняя операция меняет знак определителя. Доказательство завершено. 6. Это остается в качестве упражнения.



      Значение определителя, имеющего две одинаковые строки, равно

      Значение определителя, имеющего две одинаковые строки, равно

      Если у нас есть любая матрица с двумя одинаковыми строками или столбцами, то ее определитель равен нулю. Мы можем проверить это свойство, взяв пример матрицы A, две ее строки или столбца идентичны. Взяв пример такой матрицы, мы находим ее определитель с помощью кофакторов, и его определитель в конце оказывается равным нулю.

      Пусть определитель этой матрицы равен x, если мы поменяем местами две одинаковые строки матрицы, то по свойству определитель новой матрицы равен −x, но в целом матрица будет такой же, так как мы поменяли местами только две одинаковые строки. Итак, x = −x, имеем x = 0. Следовательно, определитель равен нулю.

      матрицы — Если в определителе есть две идентичные строки или столбцы, значение которых равно 0. Верно ли обратное? — Mathematics Stack Exchange Если в определителе есть две одинаковые строки или столбца, значение равно 0,9.0034

      Если любые две строки (или столбца) определителя одинаковы (все соответствующие элементы одинаковы), то значение определителя равно нулю. Доказательство: если поменять местами одинаковые строки (или столбцы) определителя Δ, то Δ не изменится. Однако по свойству 2 следует, что Δ сменила знак, поэтому Δ = – Δ или Δ = 0.

      Определитель любой матрицы с двумя одинаковыми строками равен 0.

      Здесь две строки R 1 и R 3 идентичны. Δ = 0. — абсолютное значение определителя (Δ), где. Известно, что площадь треугольника равна 4 квадратным единицам.

      Двумерная таблица, которая содержит следующие характеристики: Строки содержат данные об объекте, столбцы содержат данные об атрибутах объектов, все записи в столбце одного типа, каждый столбец имеет уникальное имя, ячейки таблицы содержат одно значение, порядок столбцов не имеет значения, порядок строк не имеет значения, и никакие две строки не могут быть идентичными.

      замена элементов строк и столбцов матрицы называется…

      Матричное представление — это метод, используемый компьютерным языком для хранения матриц более чем одного измерения в памяти. C использует «Главную строку», которая хранит все элементы для данной строки в памяти непрерывно.

      Определение Новая матрица, полученная путем перестановки строк и столбцов исходной матрицы, называется транспонированной матрицей.

      Родственная матричная форма, в которой строки матрицы превращаются в столбцы, а столбцы в строки, называется ____. столбцы Прежде чем вы сможете перемножить две матрицы вместе, количество ____ в первой матрице должно равняться количеству строк во второй матрице.

      Идентичные строки в матрице

      Поскольку матрица логическая, можно использовать вариант структуры данных Trie, где каждый узел будет иметь двух дочерних элементов, один для 0, а другой для 1. Вставьте каждую строку в Trie. Если строка уже существует, не печатайте строку. Если строки нет в Trie, вставьте ее в Trie и распечатайте.

      Эта задача в основном является расширением поиска уникальных строк в двоичной матрице. Простое решение состоит в том, чтобы пройти все строки одну за другой. Для каждой строки проверьте, присутствует ли она где-либо еще. Если да, распечатайте строку.

      $\begingroup$ Ваш пример просто усиливает вопрос ОП; Матрица, которую вы показываете, состоит из двух одинаковых строк, и ее определитель равен $0$. Спрашивающий хочет знать, означает ли матрица с нулями определителя, что матрица имеет две одинаковые строки или два одинаковых столбца. Ваш пример подтверждает их подозрение «Да».

      В квадратной матрице перестановка элементов строк и столбцов известна как

      Квадратная матрица — это особый тип матрицы, в которой количество строк равно количеству столбцов. Если квадратная матрица имеет $n$ строк и $n$ столбцов, говорят, что она имеет порядок $n$. Эстетически, как следует из названия, матрица выглядит как квадрат. Квадратные матрицы могут быть порядка $1$, $2$ или любого числа, $n$.

      В математике квадратная матрица — это матрица с одинаковым количеством строк и столбцов. Матрица размера n на n известна как квадратная матрица порядка n. Можно складывать и перемножать любые две квадратные матрицы одного порядка. Квадратные матрицы часто используются для представления простых линейных преобразований, таких как сдвиг или вращение.

      Если X’ — матрица, полученная путем перестановки позиций двух строк, то det (X’) = —det (x) Что называется определителями? Определитель квадратной матрицы — это значение, определяемое элементами матрицы. В матрице 2 × 2.

      докажите, что если a имеет два одинаковых столбца, то det(a)=0

      Решение для Докажите, что если A ϵ Mnxn(F) имеет два одинаковых столбца, то det(A) = 0.

      «Если det (A ) = 0, то две строки или два столбца матрицы A совпадают, или строка или столбец матрицы A равны нулю». Очевидно, что если строка или столбец A равны нулю, то при преобразовании этой матрицы в диагональную матрицу в этой строке или столбце будет ноль; это, в свою очередь, приведет к тому, что определитель (произведение диагональных элементов) будет равен нулю.

      а. Если A имеет нулевую строку (столбец), то det A 0. b. Если B получается перестановкой двух строк (столбцов) A, то det B-det A c. Если в A две одинаковые строки (столбца), то det A0 d. Если B получается умножением строки (столбца) A на k, то det B-k det A, т.е.

      Определитель перестановки строк

      1- Перестановка любых 2-х строк матрицы меняет знак ее определителя. 2- Определитель произведения 2-х матриц равен произведению определителей тех же 2-х матриц. 3- Определитель матрицы инвариантен к элементарным операциям со строками.

      Первым шагом в сокращении строк была замена строк, поэтому определитель первой матрицы отрицателен определителю второй. Таким образом, определитель исходной матрицы равен 7. Обратите внимание, что наш ответ согласуется с этим определением определителя.

      Если каждый член строк или столбцов подобен столбцу какой-либо другой строки (или столбца), то определитель эквивалентен нулю. 4.

      Если в определителе поменять местами две строки или два столбца, то полученный определитель будет отличаться только знаком. То есть, если вы поменяете местами строки или столбцы, результирующий определитель будет противоположен исходному определителю. Если вы умножаете строку или столбец на ненулевую константу, определитель умножается на ту же самую ненулевую константу.


      Вам может понравиться:

      • REGEX PLUS SIGN
      • Реализация TRIE с использованием ArrayList
      • SceneKit DAE Animation
      • Узел-красные отправки WhatsApp
      • Python Addecuation к Tpect
      • очень длинные двойные
      • . » maxlength
      • Запрос Getjson cross Origin Blocked
      • Тернарный оператор PHP
      • Cookies In Django

      Различные способы модификации определителей — Krista King Math

      Давайте поговорим о различных операциях, которые мы можем выполнять с определителем

      Теперь, когда мы поняли, что такое определитель и как его вычислить, мы хотим рассмотреть другие свойства определителей, чтобы мы могли больше с ними работать.

      Привет! Я Криста.

      Я создаю онлайн-курсы, чтобы помочь вам в учебе по математике. Читать далее.

      Умножение строки или столбца на скаляр

      Дана квадратная матрица ???A???, если умножить одну строку (или один столбец) ???A??? на скаляр ???k???, то определитель просто умножается на ???k???. Другими словами, для ???2\times2??? матрица

      определитель равен ???|A|=ad-bc???. Но если умножить любую строку ???A??? скаляром ???k???, например

      , то определитель равен ???\text{Det}(B)=|B|=k|A|??? или ???k(ad-bc)???. Неважно, какую строку вы умножите на ???k???, определитель все равно будет ???k(ad-bc)???. Это также работает для любых ???n\times n??? матрица. 92 (объявление-до н.э.)???.

      Сумма двух строк

      Когда у вас есть три одинаковые матрицы (и это, кстати, работает для любого набора ???n\times n??? матриц), за исключением того, что одна строка в каждой матрице отличается , и эта другая строка в третьей матрице является суммой различных строк из первой и второй матриц, то мы знаем, что сумма определителей первой и второй матриц равна определителю третьей матрицы.

      Например, эти матрицы ???A???, ???B??? и ???C??? у всех одинаковый первый ряд, но разные вторые ряды.

      Кроме того, вторая строка в ???C??? сумма вторых строк в ???A??? и ???Б???.

      Когда у нас есть этот конкретный случай, ???|A|+|B|=|C|???.

      ???|А|+|В|=|С|???

      ???\begin{vmatrix}1 & 0\\ 3 & 2\end{vmatrix}+\begin{vmatrix}1 & 0\\ -2 & 1\end{vmatrix}=\begin{vmatrix}1 & 0\\ 1 & 3\end{vmatrix}???

      ???[(1)(2)-(0)(3)]+[(1)(1)-(0)(-2)]=[(1)(3)-(0)( 1)]???

      ???(2-0)+(1-0)=(3-0)???

      ???2+1=3???

      ???3=3???

      И, как мы сказали, это работает одинаково для любой «другой» строки в матрицах и для любого набора таких видов ???3\times3??? или ???n\times n??? матрицы.

      Перестановка и дублирование строк

      Если поменять местами любую строку в матрице ???A??? с любой другой строкой в ​​матрице ???A???, определитель новой «матрицы перестановки строк» ​​???B??? равен отрицательному определителю ???A???. Другими словами,

      ???|В|=-|А|???

      Давайте рассмотрим простой пример. Если мы начнем с матрицы ???A???,

      затем ???B??? это матрица, которую мы получаем, когда мы переключаем строки.

      Затем, основываясь на этом правиле «перестановки строк», мы должны найти ???|B|=-|A|???. Посмотрим, получится ли.

      ???|В|=-|А|???

      ???\begin{vmatrix}3 & 2\\ 1 & 0\end{vmatrix}=-\begin{vmatrix}1 & 0\\ 3 & 2\end{vmatrix}???

      ???(3)(0)-(2)(1)=-[(1)(2)-(0)(3)]???

      ???0-2=-(2-0)???

      ???-2=-(2)???

      ???-2=-2???

      Это правило «перестановки строк», очевидно, сработало для нашего ???2\times2??? матрица, но это работает и для любых других ???3\times3??? или ???n\times n??? матрица.

      Но это правило создает нам одну проблему. Допустим, в нашей матрице есть две одинаковые строки, и мы решили поменять их местами. После того, как мы поменяем их местами, мы получим ту же матрицу, с которой начали (???A=B???), поскольку перестановка двух одинаковых строк никак не изменит матрицу.

      Что означает, что ???|A|=|B|???. Но мы сказали, что если поменять местами две строки, то ???|B|=-|A|???. Из этих двух определяющих уравнений мы говорим, что также должно быть верно, что ???|A|=-|A|???. Как это возможно? Ну только так???|А|=-|А|??? может быть правдой, если определитель равен ???0???, потому что ???0??? единственное значение, которое может удовлетворять уравнению ???x=-x???.

      Что это нам говорит? Это означает, что если любое ???n\times n??? матрица ???А??? имеет любые две одинаковые строки, то мы сразу знаем, без каких-либо вычислений, что его определитель равен ???|A|=0???, или ???|A|=0???. То же самое верно, если любое ???n\times n??? матрица ???А??? имеет любые два столбца, которые идентичны.

      И, как мы уже знаем, если определитель матрицы равен ???0???, то матрица необратима, поэтому мы можем сказать, что любое ???n\times n??? матрица с любыми двумя одинаковыми строками необратима, поэтому ее обратная не определена.

      Операции со строками не изменяют определитель

      Когда мы изучили метод исключения Гаусса для решения систем, мы узнали, как использовать операции со строками для перезаписи матрицы. Важно знать, что эти операции со строками не меняют значение определителя, если, конечно, мы не умножаем на скаляр, о чем мы говорили в начале этого урока.

      Изменение определителей путем умножения на скаляр, суммирования строк и перестановки строк

      Пройти курс

      Хотите узнать больше о линейной алгебре? У меня есть пошаговый курс для этого. 🙂

      Определение эффекта операции строки

      Пример

      Проверить, что операция строки ???R_2-3R_1\to R_2??? не изменяет значение ???|A|???.

      Если бы мы попытались представить это в виде сокращенного эшелонированного ряда, мы бы начали с ???R_2-3R_1\до R_2???. Матрица ???А??? после операций со строками будет

      Если у вас есть три одинаковые матрицы, за исключением того, что одна строка в каждой матрице отличается, а другая строка в третьей матрице является суммой различных строк из первых двух матриц, тогда мы знаем сумму определителей первой двух матриц равен определителю третьей.

      Подобные операции со строками не изменяют значение ???|A|???. Определитель ???A??? до операции строки

      И определитель ???A??? после операции строки

      В обоих случаях мы получаем одинаковое значение определителя.

      Получите доступ к полному курсу «Линейная алгебра»

      Изучение математикиКриста Кинг математика, изучение онлайн, онлайн-курс, онлайн-математика, линейная алгебра, определители, изменение определителей, умножение на скаляр, суммирование строк, суммирование строк в определителе, умножение определителя на скаляр, перестановка строк, перестановка строк в определителе, операции над строками, операции над строками в определителе, замена определителей, операции над определителями

      0 лайков

      Определитель квадратной матрицы

      6. 4 — Определитель квадратной матрицы

      Определитель — это действительное число, связанное с каждой квадратной матрицей. я еще не нашел хорошего Английское определение того, что такое определитель. Все, что я могу найти, либо определяет его в терминах математическая формула или предлагает некоторые из ее применений. Есть даже определение определитель, определяющий его через самого себя.

      Определитель квадратной матрицы A обозначается «det A» или | А |. Последнее выглядит как абсолютное значение A, но вам придется применять контекст. Если вертикальные линии расположены вокруг матрица, значит определитель.

      Строка ниже показывает два способа записи определителя.

      3 1 = от   3 1  
      5 2   5 2  

      Определитель матрицы 2×2

      Определитель матрицы 2×2 находится так же, как операция поворота. Это произведение элементов главной диагонали на минус произведение элементов вне главной диагонали.

      до н.э.
      и б = объявление —
      с д

      Свойства определителей

      • Определитель — действительное число, а не матрица.
      • Определитель может быть отрицательным числом.
      • Он вообще не связан с абсолютным значением, за исключением того, что они оба используют вертикальные линии.
      • Определитель существует только для квадратных матриц (2×2, 3×3, … п×п). Определитель матрицы 1 × 1 — это единственное значение в определителе.
      • Обратная матрица будет существовать, только если определитель не равен нулю.

      Расширение с использованием миноров и кофакторов

      Определение определителя, которое у нас есть, относится только к матрице 2×2. Существует ярлык для матрицу 3×3, но я твердо верю, что вы должны изучить способ, который будет работать для всех размеров, а не только частный случай для матрицы 3×3.

      Метод называется расширением с использованием миноров и кофакторов. Прежде чем мы сможем использовать их, мы должны определить их.

      Несовершеннолетние

      Минором для любого элемента является определитель, который получается, когда строка и столбец что элемент находится в удалении.

      Обозначение M ij используется для обозначения минора элемента в строке i и столбце j. Таким образом, M 21 будет означать минор для элемента. в строке 2, столбце 1.

      Рассмотрим определитель 3×3, показанный ниже. Я включил заголовки, чтобы вы можете оставить строки и столбцы прямыми, но обычно вы не включаете те. Мы собираемся найти некоторых несовершеннолетних.

        С 1 С 2 С 3
      Р 1 1 3 2
      Р 2 4 1 3
      Р 3 2 5 2

      Поиск минора для R

      2 С 1

      Минор — это определитель, который остается при удалении строки и столбца элемента, для которого вы пытаетесь найти минор. Это означает, что мы должны удалить строку 2 и столбец 1, а затем найти определитель.

        С 2 С 3  
      Р 1 3 2 = 3(2) — 5(2) = 6 — 10 = -4
      Р 3 5 2

      Как видите, минор для строки 2 и столбца 1 равен M 21 = -4.

      Попробуем еще.

      Поиск минора для R

      3 C 2

      На этот раз мы удалим строку 3 и столбец 2.

        С 1 С 3  
      Р 1 1 2 = 1(3) — 4(2) = 3 — 8 = -5
      Р 2 4 3

      Таким образом, минор для строки 3 столбца 2 равен M 32 = -5.

      Матрица миноров

      Когда вы просто пытаетесь найти определитель матрицы, это излишество. Но есть одно чрезвычайно полезное приложение для него, и оно даст нам практику поиск несовершеннолетних.

      Матрица миноров – это квадратная матрица, в которой каждый элемент является минором для числа в этой позиции.

      Вот общая матрица миноров для определителя 3×3.

          С 1 С 2 С 3  
      Р 1   М 11 М 12 М 13  
      Р 2   М 21 М 22 М 23  
      Р 3   М 31 М 32 М 33  

      Найдем матрицу миноров для нашего исходного определителя. Здесь определитель.

        С 1 С 2 С 3
      Р 1 1 3 2
      Р 2 4 1 3
      Р 3 2 5 2

      Вот работа по поиску каждого минора в матрице миноров.

        С 1 С 2 С 3
      Р 1
      1 3
      5 2

      = 2 — 15 = -13
      4 3
      2 2

      = 8 — 6 = 2
      4 1
      2 5

      = 20 — 2 = 18
      Р 2
      3 2
      5 2

      = 6 — 10 = -4
      1 2
      2 2

      = 2 — 4 = -2
      1 3
      2 5

      = 5 — 6 = -1
      Р 3
      3 2
      1 3

      = 9 — 2 = 7
      1 2
      4 3

      = 3 — 8 = -5
      1 3
      4 1

      = 1 — 12 = -11

      Наконец, вот матрица миноров. Опять же, вам не нужно ставить ярлыки для строки и столбцов там, но это может вам помочь.

          С 1 С 2 С 3  
      Р 1   -13 2 18  
      Р 2   -4 -2 -1  
      Р 3   7 -5 -11  

      Кофакторы

      Кофактор для любого элемента является либо минорным, либо противоположным минорному, в зависимости от того, где находится элемент в исходном определителе. Если ряд и столбец элемента в сумме должен быть четным числом, тогда кофактор — это то же, что минор. Если строка и столбец элемента в сумме являются нечетными число, то кофактор напротив минора.

      Ооо, понял? Нечетные знаки меняются, четные — это один и тот же знак. Дежавю. Мы говорили об этом начиная с раздела 3.2 о многочленах.

      Таблица знаков

      Вместо того, чтобы складывать строку и столбец элемента, чтобы увидеть, нечетное или четное, многие люди предпочитают использовать диаграмму знаков. Знаковая диаграмма либо + или — для каждого элемента в матрице. Первый элемент (строка 1, столбец 1) является всегда + и он чередуется оттуда.

      Примечание: + не означает положительное значение, а — отрицательное. + означает то же самое знак как минор, а — означает противоположность минора. Подумайте об этом дополнение и вычитание, а не положительное или отрицательное.

      Вот таблица знаков для определителя 2×2.

        С 1 С 2
      Р 1 +
      Р 2 +

      Вот таблица знаков для определителя 3×3.

        С 1 С 2 С 3
      Р 1 + +
      Р 2 +
      Р 3 + +

      Матрица кофакторов

      Опять же, если все, что вы пытаетесь сделать, это найти определитель, вам не нужно пройти через эту большую работу.

      Матрица кофакторов — это матрица, полученная заменой каждого элемента матрица своим кофактором. Это матрица миноров с измененными знаками на элементах в — позициях.

          С 1 С 2 С 3  
      Р 1   -13 -2 18  
      Р 2   4 -2 1  
      Р 3   7 5 -11  

      Расширение для поиска определителя

      Вот шаги, которые нужно выполнить, чтобы найти определитель.

      1. Выберите любую строку или столбец в матрице. Неважно, какой ряд или какой столбец, который вы используете, ответ будет одинаковым для любой строки. Есть несколько рядов или столбцы, которые проще, чем другие, но мы вернемся к этому позже.
      2. Умножить каждый элемент в этой строке или столбце по его кофактору и добавить. Результатом является определитель.

      Расширим нашу матрицу по первой строке.

      1 3 2
      4 1 3
      2 5 2

      Из таблицы знаков мы видим, что 1 находится в положительном положении, 3 — в отрицательном. положение и 2 находится в положительном положении. Ставя + или — перед элемента, он заботится о корректировке знака при переходе от минора к кофактору.

      + 1 1 3 — 3 4 3 + 2 4 1
      5 2 2 2 2 5

      = 1 (2 — 15) — 3 (8 — 6) + 2 (20 — 2)
      = 1 (-13) — 3 (2) + 2 (18)
      = -13 — 6 + 36
      = 17

      Определитель этой матрицы равен 17.

      Как я уже говорил ранее, на самом деле не имеет значения, какую строку или столбец вы используете.

      Давайте попробуем еще раз, но на этот раз расширим второй столбец. Как усилие чтобы сэкономить время, миноры для этого столбца (из матрицы миноров) были равны 2, -2 и -5. Исходными элементами были 3, 1 и 5. 3 и 5 отрицательные. позиции.

      определитель = — 3 ( 2 ) + 1 ( -2 ) — 5 ( -5 ) = -6 -2 + 25 = 17

      Разверните любую строку или любой столбец, вы получите 17.

      Но по диагонали нельзя. Если попробуем главную диагональ, получится

      + 1 (-13) + 1 (-2) + 2 (-11) = -13 -2 — 22 = -37

      Некоторые строки или столбцы лучше других

      1. Выберите строку или столбец с наибольшим количеством нулей.
        Поскольку каждый минор или кофактор умножается на элемент в матрице, выбор строки или столбца с большим количеством нулей означает, что вы будете умножение на много нулей. Умножение на ноль совсем не занимает много времени. На самом деле, если элемент равен нулю, вы не нужно найти даже несовершеннолетнего или кофактор.
      2. Выберите строку или столбец с наибольшими числами (или переменными).
        Элементы в строке или столбце, которые вы расширяете, не используются для поиска несовершеннолетние. Единственное место, где они умножаются, это один раз, в расширении. Если вы выберете строку или столбец с наименьшие числа, то каждый минор будет произведением больших чисел.
        Если вы выберете строку или столбец, в которых есть переменные, вы только имеют умножить на переменные один раз, во время расширения.

      Обратная матрица (повторно)

      На этот раз рассмотрим наш первоначальный определитель в виде матрицы.

        1 3 2  
        4 1 3  
        2 5 2  

      Найдите матрица миноров , как описано выше.

        -13 2 18  
        -4 -2 -1  
        7 -5 -11  

      Превратите его в матрицу кофакторов , изменив знаки на соответствующих элементы на основе таблицы знаков.

        -13 -2 18  
        4 -2 1  
        7 5 -11  

      Найдите сопряженное путем перестановки матрицы кофакторов.

      Чтобы транспонировать матрицу, нужно поменять местами строки и столбцы. то есть ряды стать столбцами и столбцы становятся строками. Транспонирование матрицы можно найти с помощью TI-82. или калькулятор ТИ-83, введя название матрицы и выбрав Матрица, Математика, а затем вариант 2, буква T с надстрочным индексом, например [A] Т .

        -13 4 7  
        -2 -2 5  
        18 1 -11  

      Наконец, разделите сопряженное к матрице на определитель матрицы. В этой задаче определитель равен 17, поэтому мы разделим каждый элемент на 17. Полученная матрица равна инверсия исходной матрицы.

        -13/17 4/17 17/7  
        -2/17 -2/17 17/5  
        18/17 1/17 -11/17  

      Обратная матрица находится путем деления сопряженной матрицы матрица на определитель матрицы. Не пытайтесь это сделать на своем калькулятор, так как калькулятор не позволит вам разделить матрицу на скаляр. Вместо этого вам придется умножать на обратный определитель.

      Если вы проверите это на своем калькуляторе, вы можете убедиться, что обратное на самом деле является присоединенным, деленным на определитель.

      Поскольку обратное — это сопряженное, деленное на определителя, мы можем понять, почему обратное не существует, если определитель равен нулю. Это приведет к делению на ноль, который не определен.

      Определители больших заказов

      Найдем определитель системы 4×4.

        С 1 С 2 С 3 С 4
      Р 1 3 2 0 1
      Р 2 4 0 1 2
      Р 3 3 0 2 1
      Р 4 9 2 3 1

      Выберите строку или столбец с наибольшим количеством нулей. В данном случае это второй столбец.

      Для каждого элемента в исходной матрице его минор будет определителем 3×3. Нам придется расширить каждый из них на используя три определителя 2 × 2.

      Вот почему мы хотим расширить второй столбец. Несовершеннолетние умножаются по их элементам, так что если элемент в исходной матрице равен 0, он не действительно имеет значение, что такое минор, и мы можем сэкономить много времени, не имея найти его. Во втором столбце вам не нужно будет находить двух несовершеннолетних потому что соответствующий им элемент во втором столбце равен нулю.

      — 2 4 1 2 + 0       — 0       + 2 3 0 1
      3 2 1   ?     ?   4 1 2
      9 3 1             3 2 1

      Мы могли бы заполнить эти средние два минора, но так как они умножаются на 0, на самом деле не имеет значения, что они собой представляют. На самом деле, вы могли бы так же легко пропустить их.

      Теперь осталось найти два определителя 3×3.

      В первом определителе 3×3, нулей нет, поэтому выберите строку или столбец с наибольшими числами. Что будет столбец 1, поэтому расширьте его по первому столбцу.

      Уведомление 4 находится в положительном положении. Таблицы знаков начинаются с каждого новый определитель. Положение числа в исходной матрице не имеет значение только его положение в текущей матрице.

      4 1 2                    
      3 2 1 = + 4 2 1 — 3 1 2 + 9 1 2
      9 3 1     3 1   3 1   2 1

      = 4 (2 — 3) — 3 (1 — 6) + 9 (1 — 4) = 4 (-1) — 3 (-5) + 9 (-3) ) = -4 + 15 — 27 = -16

      Рассмотрим другую матрицу 3×3. В этой строке стоит 0 1 и столбец 2. Любой из них был бы хорошим выбором для расширения, но поскольку в строке 1 числа немного больше, мы будем расширяться по первой строке.

      3 0 1                    
      4 1 2 = + 3 1 2 — 0 ? ? + 1 4 1
      3 2 1     2 1   ? ?   3 2

      = 3 (1 — 4) — 0 (не имеет значения) + 1 (8 — 3) = 3 (-3) + 1 (5) = -9 + 5 = -4

      Когда пойдете искать определитель, помните, что там были элементы из исходная матрица 4 × 4, умноженная на каждый из этих определителей 3 × 3. Первый был -2, а второй +2.

      Определитель = -2 (-16) + 2 (-4) = 32 — 8 = 24

      Наихудший сценарий

      Чтобы найти определитель 3×3 без нулей, нужно найти три определителя 2×2.

      Чтобы найти определитель 4×4 без нулей, нужно найти четыре определителя 3×3, каждый из которых затем становится тремя определителями 2×2, всего получается двенадцать определителей 2×2.

      Чтобы найти определитель 5×5 без нулей, нужно найти пять определителей 4×4, каждый из которых затем становится четырьмя определителями 3×3, каждый из которых становится тремя определителями 2×2 в сумме из шестидесяти определителей 2×2.

      Использование калькулятора

      После этой последней проблемы вы должны спросить себя, нет ли более легкого пути. Ну да, есть, если в определителе нет переменных. Вы можете воспользоваться калькулятором.

      В калькуляторах TI-82 или TI-83 используются обозначения Det A. Итак, после входа в матрицу в одну из доступных матриц на калькуляторе, введите DET, выбрав Матрица, Математика и выбрав вариант 1. Затем введите имя матрицы, которую вы используете.

      Вам не нужно использовать круглые скобки (если у вас нет TI-83), но вы можете, если вы хотите найти определитель произведения «det ([A]*[B])» или определитель транспонированного «det ([A] T )» как в отличие от транспонирования определителя «(det [A]) . Кстати, калькулятор не найдет транспонирование определителя, потому что в определитель является скаляром (действительным числом) и калькулятор знает только, как найти транспонирование матрицы. Транспонирование скаляр это что скаляр.

      Треугольные матрицы

      Вам очень понравится находить определители этих матриц.

      Верхняя треугольная матрица
      Матрица, в которой все ненулевые элементы находятся либо на главной диагонали, либо над ней. То есть все ненулевые значения находятся в верхнем треугольнике. Все, что ниже диагонали является нулем.
      Нижняя треугольная матрица
      Матрица, в которой все ненулевые элементы находятся либо на главной диагонали, либо ниже нее.
      То есть все ненулевые значения находятся в нижнем треугольнике. Все, что выше диагонали равен нулю.
      Диагональная матрица
      Матрица, в которой все ненулевые элементы находятся на главной диагонали. Все выключено главная диагональ равна нулю.

      Определитель треугольной матрицы или диагональной матрицы равен произведению элементов главной диагонали.

      Элементарные операции со строками

      Можно было выполнить три элементарные операции со строками, которые вернули бы эквивалентная система. С определителями, поскольку определитель транспонирования совпадает с определителя матрицы, элементарные операции со строками можно применять и к столбцам.

      Выполняя сокращение строк (используя поворот на 1, если хотите), вы можете поместить матрицу в треугольная форма. Как только он будет в треугольной форме, все, что вам нужно сделать, это умножить на элементы на главной диагонали и у вас есть определитель.

      Рассмотрим каждую из трех элементарных операций над строками.

      1. Если поменять местами две строки или два столбца в определителе, результирующий определитель отличаются только знаком. То есть, если вы поменяете местами строки или столбцы, результирующий определитель будет напротив исходного определителя.
      2. Если вы умножаете строку или столбец на ненулевую константу, определитель умножается на эту та же ненулевая константа.
      3. Если вы умножаете строку или столбец на ненулевую константу и добавляете ее к другой строке или столбцу, заменив эту строку или столбец, определитель не изменится.

      Эта последняя операция эквивалентна повороту на единицу!

      Предупреждение: если ваша точка опоры — это число, отличное от единицы, то вы умножаете каждую строку, которую вы изменение поворотным элементом. Итак, если вы повернетесь на 3 и поменяете две строки, то в результате определитель будет 3 * 3 = 9раз больше исходного определителя.

      Пока вы вращаетесь на одном, все будет в порядке.

      Вам не нужно помещать матрицу в редуцированную строчно-эшелонную форму или даже в строчно-эшелонную форму. Вы можете остановить сокращение в любой момент и расширить его, используя миноры и кофакторы. Что я предложить является сводным, где есть один, а затем расширить.

      Определители, равные нулю

      Определитель матрицы будет равен нулю, если

      1. Вся строка равна нулю.
      2. Две строки или столбца равны.
      3. Строка или столбец постоянно кратны другой строке или столбцу.

      Помните, что матрица обратима, неособа, тогда и только тогда, когда определитель не равен нулю. Итак, если определитель равен нулю, матрица вырожденная и не имеет обратной.

      Детерминанты: Определение

      Цели
      1. Изучите определение определителя.
      2. Научитесь находить на глаз матрицу с нулевым определителем и вычислять определители матриц верхнего и нижнего треугольников.
      3. Изучите основные свойства определителя и способы их применения.
      4. Рецепт: вычислить определитель, используя операции со строками и столбцами.
      5. Теоремы: теорема существования, свойство обратимости, свойство мультипликативности, свойство транспонирования.
      6. Словарные слова: диагональные , верхнетреугольные , нижнетреугольные , транспонированные .
      7. Основное словарное слово: определитель .

      В этом разделе мы определяем определитель и представляем один из способов его вычисления. Затем мы обсудим некоторые из многих замечательных свойств, которыми обладает определитель.

      Определитель квадратной матрицы A является вещественным числом det(A). Он определяется своим поведением по отношению к операциям со строками; это означает, что мы можем использовать сокращение строк для его вычисления. Мы дадим рекурсивную формулу для определителя в разделе 4. 2. В этом подразделе мы также покажем, что определитель связан с обратимостью, а в разделе 4.3 — с объемами.

      Определение

      Определитель является функцией

      det:CsquarematrixD−→R

      , удовлетворяющий следующим свойствам:

      1. Выполнение замены строки в A не меняет det(A).
      2. Масштабирование строки A с помощью скаляра c умножает определитель на c.
      3. Замена двух строк матрицы умножает определитель на -1.
      4. Определитель единичной матрицы In равен 1.

      Другими словами, каждой квадратной матрице A мы присваиваем число det(A) способом, который удовлетворяет указанным выше свойствам.

      В каждом из первых трех случаев выполнение операции над строкой матрицы масштабирует определитель на ненулевое число. (Умножение строки на ноль не является операцией со строками.) Следовательно, выполнение операций со строками над квадратной матрицей A не влияет на то, равен ли определитель нулю.

      Основная мотивация использования этих определяющих свойств носит геометрический характер: см. раздел 4.3. Еще одна причина для этого определения заключается в том, что оно говорит нам, как вычислить определитель: мы уменьшаем количество строк и отслеживаем изменения.

      Пример

      Вычислим detA2114B. Сначала мы уменьшаем строку, затем вычисляем определитель в обратном порядке:

      M2114Ndet=7R1 ←→R2——-→M1421Ndet=-7R2=R2-2R1——-→M140-7Ndet=-7R2=R2÷-7——-→ M1401Ndet=1R1=R1-4R2——-→M1001Ndet=1

      Сокращенная эшелонная форма строки матрицы представляет собой единичную матрицу I2, поэтому ее определитель равен 1. Предпоследним шагом в сокращении строки была замена строки, поэтому вторая конечная матрица также имеет определитель 1. Предыдущий шаг в сокращение строки было масштабированием строки на -1/7; поскольку (определитель второй матрицы, умноженный на -1/7) равен 1, определитель второй матрицы должен быть равен -7. Первым шагом в сокращении строк была замена строк, поэтому определитель первой матрицы отрицателен, как определитель второй. Таким образом, определитель исходной матрицы равен 7,9.0034

      Обратите внимание, что наш ответ согласуется с этим определением определителя.

      Пример
      Пример

      Вот общий метод вычисления определителей с использованием редукции строк.

      Рецепт: вычисление определителей путем редукции строк

      Пусть A — квадратная матрица. Предположим, что вы выполняете некоторое количество операций со строками над A, чтобы получить матрицу B в форме эшелона строк. Затем

      det(A)=(−1)r·(произведение диагональных элементов B)(произведение использованных коэффициентов масштабирования),

      , где r — количество выполненных перестановок строк.

      Другими словами, определитель A является произведением диагональных вхождений формы эшелона строк B, умноженных на коэффициент ±1, полученный из числа сделанных вами перестановок строк, деленное на произведение коэффициентов масштабирования, используемых в сокращение ряда.

      Пример
      Пример
      Пример (Определитель матрицы 2 × 2)

      Воспользуемся рецептом для вычисления определителя общей матрицы 2×2 A=AabcdB.

      • Если а=0, то

        detMabcdN=detM0bcdN=-detMcd0bN=-bc.

      • Если аВ=0, то

        detMabcdN=a·detM1b/acdN=a·detM1b/a0d-c·b/aN=a·1·(d-bc/a)=ad-bc.

      В любом случае мы восстанавливаем формулу из раздела 3.5:

      detMabcdN=ad-bc.

      Если матрица уже имеет форму эшелона строк, то вы можете просто считать определитель как произведение диагональных элементов. Оказывается, это верно для немного большего класса матриц, называемого треугольный .

      Определение
      • диагональных элементов матрицы A — это элементы a11,a22,…:a11a12a13a14a21a22a23a24a31a32a33a34GKIHLJa11a12a13a21a22a23a31a32a33a41a42a43GKKKKIHLLries
      • Квадратная матрица называется верхнетреугольной , если все ее ненулевые элементы лежат выше диагонали, и называется нижнетреугольной , если все ее ненулевые элементы лежат ниже диагонали. Называется диагональ , если все его ненулевые элементы лежат на диагонали, т. е. если он одновременно является верхнетреугольным и нижнетреугольным. AAAA0AAA00AA000AGKKKIHLLLJверхний треугольныйA000AA00AAA0AAAAGKKKIHLLLJнижний треугольныйA0000A0000A0000AGKKKIHLLLJдиагональный
      Предложение

      Пусть A — матрица размера n × n.

      1. Если A имеет нулевую строку или столбец, то det(A)=0.
      2. Если A является верхнетреугольным или нижнетреугольным, то det(A) является произведением его диагональных элементов.
      Доказательство
      1. Предположим, что A имеет нулевую строку. Пусть B — матрица, полученная инвертированием нулевой строки. Тогда det(A)=-det(B) по второму определяющему свойству. Но A=B, поэтому det(A)=det(B):

        E123000789FR2=-R2—-→E123000789F.

        Если сложить их вместе, получится det(A)=−det(A), поэтому det(A)=0.

        Теперь предположим, что A имеет нулевой столбец. Тогда A необратима по теореме об обратимой матрице из раздела 3.6, так что ее редуцированная ступенчатая форма строк имеет нулевую строку. Поскольку операции со строками не меняются, если определитель равен нулю, мы заключаем, что det(A)=0.

      2. Сначала предположим, что A является верхнетреугольным и что один из диагональных элементов равен нулю, скажем, aii=0. Мы можем выполнять операции со строками, чтобы очистить записи над ненулевыми диагональными записями:

        GKIa11AAA0a22AA000A000a44HLJ−−−→GKIa110A00a22A00000000a44HLJ

        В результирующей матрице i-я строка равна нулю, поэтому det(A)=0 по первой части.

        По-прежнему предполагая, что A является верхнетреугольным, теперь предположим, что все диагональные элементы A отличны от нуля. Затем A можно преобразовать в единичную матрицу, масштабируя диагональные элементы и затем выполняя замену строк:

        EaAA0bA00cFscalebya-1,b-1,c-1——-→E1AA01A001Frowreplacements——-→E100010001Fdet=abc ←———det=1 ←——- −det=1

        Поскольку det(In)=1 и мы масштабировали по обратным величинам диагональных элементов, отсюда следует, что det(A) является произведением диагональных элементов.

        Тот же аргумент работает для нижних треугольных матриц, за исключением того, что замены строк идут вниз, а не вверх.

      Пример

      Матрица всегда может быть преобразована в ступенчатую форму с помощью серии операций над строками, а матрица в ступенчатой ​​форме строк является верхнетреугольной. Таким образом, мы полностью оправдали рецепт вычисления определителя.

      Определитель характеризуется своими определяющими свойствами, поскольку мы можем вычислить определитель любой матрицы, используя сокращение строк, как в приведенном выше рецепте. Однако мы еще не доказали существование функции, удовлетворяющей определяющим свойствам! Сокращение строк будет вычислять определитель , если он существует , но мы не можем использовать сокращение строк, чтобы доказать существование, потому что мы еще не знаем, что вы вычисляете одно и то же число путем сокращения строк двумя разными способами.

      Теорема (Существование определителя)

      Существует одна и только одна функция из набора квадратных матриц для действительных чисел, которая удовлетворяет четырем определяющим свойствам.

      Мы докажем теорему существования в разделе 4.2, представив рекурсивную формулу для определителя. Опять же, реальное содержание теоремы существования таково:

      Независимо от того, какие операции со строками вы выполняете, вы всегда будете вычислять одно и то же значение для определителя.

      В этом подразделе мы обсудим ряд удивительных свойств, которыми обладает определитель: свойство обратимости, свойство мультипликативности и свойство транспонирования.

      Свойство обратимости

      Квадратная матрица обратима тогда и только тогда, когда det(A)B=0.

      В силу свойства обратимости определитель матрицы, не удовлетворяющей ни одному из свойств теоремы об обратимой матрице из раздела 3.6, равен нулю.

      Следствие

      Пусть A — квадратная матрица. Если строки или столбцы A линейно зависимы, то det(A)=0.

      В частности, если две строки/столбца матрицы A кратны друг другу, то det(A)=0. Мы также восстанавливаем тот факт, что матрица со строкой или столбцом нулей имеет нулевой определитель.

      Пример

      Доказательства свойства мультипликативности и свойства транспонирования ниже, а также теоремы о разложении кофакторов в разделе 4.2 и теоремы об определителях и объемах в разделе 4.3 используют следующую стратегию: определить другую функцию d: {n×nmatrices}→R , и докажите, что d удовлетворяет тем же четырем определяющим свойствам, что и определитель. По теореме существования функция d равна определителю . В этом преимущество определения функции через ее свойства: чтобы доказать, что она равна другой функции, нужно только проверить определяющие свойства.

      Свойство мультипликативности

      Если A и B матрицы размера n×n, то

      det(AB)=det(A)det(B).

      Напомним, что взять степень квадратной матрицы A означает взять произведение A на себя:

      A2=AAA3=AAAи т.д.

      Если A обратим, то мы определяем

      А-2=А-1А-1А-3=А-1А-1А-1 и т. д.

      Для полноты мы устанавливаем A0=In, если AB=0.

      Следствие

      Если A — квадратная матрица, то

      дет(Ан)=дет(А)n

      для всех n≥1. Если A обратим, то уравнение верно и для всех n≤0; в частности,

      det(A−1)=1det(A).
      Пример

      Вот еще одно применение свойства мультипликативности.

      Следствие

      Пусть A1,A2,…,Ak — матрицы размера n × n. Тогда произведение A1A2···Ak обратимо тогда и только тогда, когда обратимо каждое Ai.

      Доказательство

      Определитель произведения равен произведению определителей по свойству мультипликативности:

      det(A1A2···Ak)=det(A1)det(A2)···det(Ak).

      По свойству обратимости это не равно нулю тогда и только тогда, когда A1A2···Ak обратим. С другой стороны, det(A1)det(A2)···det(Ak) отличен от нуля тогда и только тогда, когда каждый det(Ai)B=0, что означает, что каждый Ai обратим.

      Пример

      Для определения свойства транспонирования нам необходимо определить транспонирование матрицы.

      Определение

      Транспонирование матрицы A размера m×n представляет собой матрицу размера n×m AT, строки которой являются столбцами матрицы A. Другими словами, элемент ij матрицы AT равен aji.

      a11a12a13a14a21a22a23a24a31a32a33a34GKKIHLLJAa11a21a31a12a22a32a13a23a33a14a24a34GKKKKKIHLLLLLJATBip

      Как и инверсия, транспозиция меняет порядок умножения матриц на обратный.

      Факт

      Пусть A — матрица размера m×n, а B — матрица размера n×p. Затем

      (АВ)Т=БТАТ.

      Транспонировать свойство

      Для любой квадратной матрицы A мы имеем

      дет(А)=дет(А).

      Свойство транспонирования очень полезно. Отметим для конкретности, что det(A)=det(AT) означает, например, что

      detE123456789F=detE147258369F.

      Это означает, что у определителя есть любопытная особенность, заключающаяся в том, что он также хорошо ведет себя по отношению к операциям со столбцом . В самом деле, операция со столбцом над A аналогична операции над строкой над AT, и det(A)=det(AT).

      Следствие

      Определитель удовлетворяет следующим свойствам в отношении операций со столбцами:

      1. Замена столбца в A не меняет det(A).
      2. Масштабирование столбца A скаляром c умножает определитель на c.
      3. Замена двух столбцов матрицы умножает определитель на −1.

      Предыдущее следствие упрощает вычисление определителя: при упрощении матрицы можно выполнять строк и операций со столбцами. (Конечно, нужно еще следить за тем, как операции со строками и столбцами изменяют определитель.)

      Пример
      Мультилинейность
      Резюме: Магические свойства определителя
      1. Существует одна и только одна функция det:{n×nmatrices}→R, удовлетворяющая четырем определяющим свойствам.
      2. Определитель верхнетреугольной или нижнетреугольной матрицы является произведением диагональных элементов.
      3. Квадратная матрица обратима тогда и только тогда, когда det(A)B=0; в таком случае, det(A−1)=1det(A).
      4. Если A и B матрицы размера n×n, то

        det(AB)=det(A)det(B).

      5. Для любой квадратной матрицы A имеем

        дет(АТ)=дет(А).

      Добавить комментарий

      Ваш адрес email не будет опубликован. Обязательные поля помечены *